Anda di halaman 1dari 54

Chapter 2 obtained from heating.

When using
Physical Therapy Modalities hyperemia as the index of the
therapeutic response, the range of
Directions: Each of the questions or incomplete duration of temperature elevation
statements below is followed by five suggested that should be maintained to
answer or completions. Selects the BEST answer in achieve maximal effectiveness is D
each case. A. 1
B. 1 to 3 min
1. the physiological effect of local C. 5 to 120 min
heating include: E D. 3 to 30 min
A. increased local metabolic rate E. none of the above.
B. increased local vasodilation 6. Which of the techniques listed
C. increased local capillary hydrostatic below is most effective in
pressure producing permanently improved
D. local analgesic flexibility? A
E. all of the above A. prolonged stretching during heating
with release from stretching after
2. Which of the following is not a the tissue has cooled to normal
therapeutic effect of vigorous local temperature.
heating?B B. Short duration stretching whole the
A. increased blood flow due to tissue is at maximum temperature
arteriolar and capillary dilation C. prolonged stretching during
B. steady increase in enzyme activity heating, with released from
as the temperature is elevated. stretching before the tissue cools to
C. Increase in the rate of diffusion normal tissue temperature
across biologic membranes D. heating the tissue to maximum
D. Increase in the extensibility of temperature, then prolonged
fibrous connective tissue. stretching then cooling to the
E. Decreased viscoeleastic resistance normal tissue temperature without
to joint stretching
E. heating actually does not improve
3. hyperemia is the most easily the results permanently when used
measured effect of local heating. It with stretching to gain tissue
is often used in research studies as flexibility.
an indication of the response of
tissue to heating. Hyperemia first 7. Which of the following offers the
occurs at what temperature? C quickest response to actual blood
A. 1C above the normal core temperature? A
temperature A. oral temperature
B. 37C (98.6F) B. rectal temperature
C. 43C (109.4F) C. axillary temperature
D. 45C (113F) D. skin temperature over the trunk
E. none of the above E. skin temperature over the
extremities
4. Using hyperemia as the indicator of
therapeutic effect, the therapeutic 8. Which of the following would be
temperature range isC the most likely to elevate core
A. 43C to 49C temperature? (assume that the
B. 37C to 43C patient has normal circulatory
C. 43C to 45C mechanisms) A
D. 45C to 50C A. trunk and extremities in Hubbard
E. 0C to 100C tank or pool at 38.8C
B. hot air bath (saturated with water
5. the Duration of the elevated tissue vapor) at 45C
temperature is an important factor C. paraffin dip-wrap of one hand and
in the degree of biologic response wrist at 52C
D. ultrasound at 1.5 w/cm2 to the 1. counter irritation theory (attributed to
lateral epicondyle 1 or 7 mm hippocrates)
E. short wave diathermy to the lumbar 2. gate theory
spine for 20 min 3. thermal gradient equalization theory of
wells
9. The maximum safe exposure of the 4. reduction in peripheral nerve integrity
skin to heated water is 45C for 30 theory
min. paraffin dipwrap coats the skin
with a mixture of mineral oil (one 13. Heat can be transferred by :A
part) and paraffin (seven parts) at a 1. conduction
temperature of 52C. why does the 2. convection
skin not get burned.D 3. radiation
A. protective subcutaneous fat layer 4. kinetic energy
B. rapid skin vasodilation
C. paraffin changes rapidly from a 14. The significant factors in
liquid to a solid on contact with determining the quantity of
skin radiation a patient receives are A
D. paraffin has a lower thermal 1. distance between the source and the
conductivity than water patient
E. none of the above. 2. wavelength of the radiation
3. angle of incidence of the radiation to the
Directions: for each incomplete statement below. skin surface
ONE or MORE of the numbered completions is 4. color of the skin
correct. In each case select:
A. if only 1,2 and 3 are correct 15. Converse hearing refers to :D
B. if only 1 and 3 are correct 1. heating that involves conduction
C. if only 2 and 4 are correct 2. healing that involves changing from
D. if only 4 correct potential energy to thermal energy
E. if all are correct 3. heating by direct application
4. the use of heating modality that passes
10. The correct formula(s) for through superficial tissue and is then
converting Fahrenheit degrees to changes to heat in the deeper tissues
centigrade (Celsius) degrees (are):
C 16. The heat transition is: B
1. Fahrenheit degrees plus 273 1. the head gained or lost when a
2. C = 5/9 (F – 32) substance to conduct heat
3. C = F + 32 2. the natural tendency of a substance to
4. F = 9/ 5C + 32 conduct heat
3. more than the amount of heat needed to
11. Local heating is contraindicating raise a substance one degree without a
(or at least must be used with change in physical state.
extreme caution) in patients with 4. The term used to describe the
1. vascular insufficiency relationship of the temperature of a
2. impaired pain and temperature sensation substance to the radiation it emits.
3. edema
4. senility, disorientation or any other 17. Wien’s Law is: D
condition reducing the patient’s ability 1. that some patients react negatively to
to rationally cooperate with the the use of modalities
treatment 2. a mathematical statement of the ability
of a substances to conduct heat
12. Heat has an analgesic effect (which 3. a statement of the relationship of the
explains why it hurts less to catch a temperature of a body and the quantity
ball with bare hands on a warm day of radiation in emits
than on a cold day) The current 4. a statement of the relationship of the
theories explaining this analgesic temperature of a body and the
effect include the :A wavelength of radiation it
2. temporary reduction of spasticity
18. Specific heat is the : C 3. retarding the development of gangrene
1. tendency of a substance to conduct heat in an ischemic limb
2. amount of heat a substance actually 4. improving the blood supply to an
contains at a given temperature ischemic limb
3. type of heat being given (for example,
ultrasound or diathermy) 23. Spasticity in some patient can be
4. reason why some substance take more temporarily reduced by the local
heat than others to raise their cooling of the affected muscles.
temperature one degree centigrade. The most likely reason(s) for this
effect is (are): A
19. The Stefan-Boltzman law states 1. muscle spindle afferent impulse firing
that :D decrease
1. higher temperatures of a substance 2. cooling reduces the membrane
cause shorter wavelength of radiation excitability of axons
emission 3. gamma motor fibers are more easily
2. patients with pacemakers should not be blocked by cold than alpha motor fibers
exposed to microwave radiation 4. small and unmyelinated nerve fibers are
3. thermal conductivity is higher for water more resistant to blockage by cold than
than for paraffin –mineral oil other nerve fiber types.
4. the quantity of radiation emitted by an
object is proportion to the fourth power 24. Superficial local cold penetrates
of the temperature. deeper through human tissue than
superficial local heat because: B
20. Thermal conductivity: A 1. arteriolar vasoconstriction occurs with
1. determine how hot or cold an object feel cold
when placed against the skin 2. arteriolar vasodilation occurs with cold
2. is the natural tendency of a substance to 3. the countercurrent heat exchange
conduct heat (or cold) system keeps blood in a cooled
3. is lower for wood than for metal such as extremity
copper and iron 4. nerve conduction decreases with cooler
4. determines the quality of heat a temperatures
substance actually contains at a given
temperature per unit mass. 25. The important consideration(s) in
treating patients in the Hubbard
21. A daily hand program for the tank is (are) : E
patient with rheumatoid arthritis 1. patients will overheat is emerged in
often includes a paraffin dip wrap water above 38.8C
followed by prescribed exercise of 2. patients with extensive burns should be
the wrist and hands. The most treated in a physiological saline solution
convenient technique(s) for paraffin rather than in water
dip-wrap in the home is (are): D 3. great care must be exercised to prevent
1. melting of the paraffin mineral oil cross infection of patients receiving
mixture each day in a deep pan tank therapy
2. melting of the paraffin mineral oil 4. patients with congestive heart failure
mixture each day in a double boiler may not be able to tolerate generalized
3. keeping the paraffin mineral oil mixture body heating
at the appropriate temperature at all 26. The major advantage of the Hubbard tank
times a double boiler for hydrotherapy is (are): C
4. keeping the paraffin mineral oil mixture 1. it is easily sterilized in a few
melted and at the correct temperature in minutes between patients
a specially designed thermostatically 2. it allows full range of motion of the
controlled home unit. extremities under water
3. it is large enough to allow the
22. Local cooling is indicated for: A therapist to get into the tank with
1. reduction of posttraumatic edema
the patient for range of motion 2. a competitive pacemaker is the type most
therapy likely to be affected by short-wave
4. it has a figure-eight shape that diathermy
allows the therapist to reach the 3. microwave diathermy affects pacemakers,
patient adequately without having but short wave diathermy does not
to get into the tank 4. microwave and short wave diathermy
pose a serious danger for patients using
27. The beneficial effects of hydrotherapy any type of implanted cardiac pacemaker
include : E
1. buoyancy effect on body parts 40. In selecting the dosage of diathermy given to
2. mechanical cleansing of the skin due to the patient, the therapist monitors the: D
the agitation of the water 1. skin temperature of the area being treated
3. analgesic effect of warm water 2. power meter of the diathermy machine
4. the posttraumatic antiedema effect of 3. careful selection of a resonance frequency
cold water between the oscillating circuit of the
machine and the patient circuit
28. Hydrocollator packs heated at 60C to 70C 4. patient’s sensation of warmth in the area
are used instead of the towels or hot water being treated
bottle in most physical therapy departments
because: C 41. The federal Communication Commission
1. are less expensive to use granted the use of the 2,459 MHz band of the
2. maintain heat longer than hot microwave spectrum to medical use because:
towels made of cotton or wool D
3. can be heated to a higher 1. this frequency is capable of producing
temperature than hot towels deeper heating of tissues than other
4. are convenient and sanitary as frequencies
compared to hot towels 2. it is more expensive to produce machines
Directions: For each of the incomplete statements with higher or lower frequencies
below, ONE or MORE of the numbered completion is 3. lower frequencies interfere more with
correct. Each case select: military radar
A. if only 1,2 and 3 are correct 4. generators producing this frequency were
B. if only 1 and 3 are correct available when microwave therapy was
C. if only 2 and 4 are correct introduced into practice after World War II
D. if only 4 is correct
E. if all are correct 42. The apparatus which produces microwaves for
clinical purpose:E
37. Diathermy literally means “heat through” The 1. generated the microwaves by application
clinically used heat modalities generally of voltage to a multicavity magnetron tube
referred to as diathermy include:A 2. is similar to microwave ovens and radar
1. short wave apparatus except in the frequency of the
2. ultrasound microwaves generated
3. microwave 3. has a frequency of 2,450 mega Hertz
4. ultraviolet assigned by the Federal Communications
Commission
38. The frequencies granted by the Federal: A 4. uses a director or antenna to apply the
1. 13.66 MHz (wave length 22mm) microwave to the patient
2. 27.33 MHz ( wave length 11mm)
3. 40.98 MHz ( wave length 7.5 m) 43. The tissues that best absorb microwaves are :D
4. 2450 MHz (wave length 12.2 cm) 1. bone
2. cartilage
39. Which of the following statements regarding 3. hair
diathermy and cardiac pacemakers is (are) 4. tissue containing the highest percentage of
true?): A water such as muscle fat, and eye humors
1. a nociceptive pacemaker presents no
significant hazard or diathermy use 44. Which of the following are important in
administering microwave therapy? E
1. preventing sweat droplets from forming 1. produces a pencil thin beam emanating
by placing towels on the area to be treated from the center of the applicator
2. placing the microwave director no closer 2. produces a beam with a broad based
than 2cm from the skin surface intensity distribution curve across the
3. not exposing the eyes to vigorous full diameter of the applicator
treatment with microwaves 3. has large applicator surface (greater
4. using multiple sites to heat a joint than 15cm2)
4. has an applicator surface large enough
45. Which of the following are indications for to have a low angle of divergence of the
microwave therapy? B sound beam, but small enough to
1. superficial inflammatory reactions such as maintain contact with uneven body
furuncles and abscesses surfaces
2. the need for vigorous heating of the
internal structures of the hip joint 66. Ultrasound machines for clinical therapy have a
3. the need for heating of the finger joints frequency of : D
4. protruded intervertebral disc syndrome 1. 17,000 Hz
2. 5,000 Hz
46. Microwave intensities commonly used in 3. 500,000 Hz
clinical physical therapy are D 4. 100,000 Hz
1. not of sufficient intensity to produce any
lens cataracts 67. Ultrasound produces heat in tissues by particle
2. safe for use over fluid collections movement within the field of the sound beam. The
including effusions and edematous tissues acceleration of these particles with the standard
3. safe for use over metallic implants clinical ultrasound machine is about :D
4. usually 0.1 to 1.0 w/cm2 when measured 1. 1 to 4 atmospheres of pressure
with special equipment 2. 1.5 g
3. 3.0 g
47. The most serious problem(s) affecting the eye 4. 100,000 g
with the use of microwave is (are): C
1. selective heating of the vitreous humor 68. Ultrasound beams used for clinical therapy do not
2. selective heating of the aqueous humor have a uniform field of sound intensity. The specific
3. the formation of lens cataracts characteristic of the sound beam include the : E
4. the development of glaucoma 1. “near field” being the are between the
translucent and the most distant
Directions: For each of the incomplete statements intensity maximum of sound.
below, ONE or MORE of the numbered 2. “near field” having much greater
completions is correct. In each case select. variation sites than does the “far field”
A. if only 1,2 and 3 are correct 3. ‘near field” extending for many
B. if only 1 and 3 are correct centimeters in clinical machines
C. if only 2 and 4 are correct 4. “far field” having a fairly uniform
D. if only 4 is correct intensity of sound
E. if all are correct
69. Ultrasound intensity is measured in W cm2. this
64. Ultrasound is generated for clinical purposes: refers to the:D
B 1. watts of sound centimeters per square
1. by a reversal of the piezoelectric centimeter of skin surface
effect 2. watts of electricity used to generate the
2. by a small paper cone speaker sound per square centimeter of skin that
3. by a radio frequency generator is treated
which produces an oscillating 3. total watts of electricity used to generate
circuit to drive the transducer the sound beam per centimeter of depth
4. by a standard electrical vibrator of penetration in the tissues
4. total output of tye applicator in watts
65. The most appropriate transducer applicator divided by the total surface area of the
for clinical use is one that :C applicator
70. The wavelength of ultrasound is : C 3. ultrasound, infrared, short wave
1 such that some humans can hear it diathermy
2. much shorter in tissue than the wave 4. ultrasound, short wave diathermy,
length of sound produced by the human infrared.
voice
3 the same in the air as it is in tissue Directions: For each of the incomplete statements
4. such that a layer of maternal tends to below, ONE or MORE of the numbered completion is
reflect or absorb it if the thickness of the correct. Each case select:
layer is equal to or greater than the A. if only 1,2 and 3 are correct
wavelength of ultrasound B. if only 1 and 3 are correct
C. if only 2 and 4 are correct
71. The wavelength of ultrasound: B D. if only 4 is correct
1. is about 1 mm in tissue E. if all are correct
2. is shorter in tissue than in air
3. changes in tissue because sound travels 152. Ultraviolet effects include :C
faster in tissue than in air 1. tanning of the skin, which then helps
4. is greater in vaccum protect from further ultraviolet injury
2. thickening of the stratum comeum, which
72. The pattern of relative heating in ultrasound is protects from further ultraviolet injury
such that: B 3. tanning that occurs as a result of a broad
1. a large joint must be heated from all band of wavelength from 2,000 A to 4,000
with A.
2. ultrasound penetrated bone better than 4. injury of the prickle cell layers of skin,
soft tissue resulting in the release of vasodilators that
3. ultrasound heats the deep tissues with subsequently produce the erythematous
relatively little hearing of skin and changes
subcutaneous tissue
4. ultrasound heats tissue no deeper than 153. Excessive exposure to ultraviolet light may
the depth achieved with short wave cause:B
diathermy 1. squamous cell and basal cell skin
carcinoma
73. Reflection of ultrasound: E 2. severe erythema if the ultraviolet is
1. occurs mainly at interfaces of tissues transmitted through glass plates
with different acoustic impedances 3. marked thickening and loss of skin
2. is great at the interface of soft tissue and pliability
bone 4. permanent tanning in Caucasians
3. is great at the interface of soft tissue and
metallic implants 154. Excessive exposure to ultraviolet produces eye
4. is minimal at interfaces of layers of soft complications such as :E
tissue 1. conjunctivitis
2. corneal keratotic changes
74. Absorption of ultrasound: E 3. lenticular opacities and cataracts
1. is greater in soft tissues with high 4. pain
proteins content
2. occurs to some extent at interfaces of 155. infrared radiation in the far infrared range
different soft tissue layers (14,000-400,00 A) penetrated to about 0.1 mm while
3. is very high in bone near infrared radiation (7,700 – 14,000A) penetrates
4. can be calculated for each tissue type to 3 mm. patients usually have a greater feeling of
(often called the ultrasound tissue warmth from the far infrared because.: D
absorption) coefficient) 1. the near infrared is partially in the visible
spectrum
75. The order of deepest penetrates of these heat 2. the near infrared is closet to ultraviolet the
modalities is : D electromagnetic spectrum
1. short wave diathermy, ultrasound 3. far infrared healers give a more uniform
infrared radiation pattern
2. infrared, short wave, ultrasound
5. far infrared more effectively heats the
nerve ending in the skin 161. Patients with painful back conditions often
improve when lumbar traction is used continuously
156. The best source of infrared radiation is:B for many hours with distractions weights of 50
1. an ordinary frosted incandescent light pounds or less. This is because :B
bulb 1. the bed rest the patient gets while in the
2. a red-filtered incandescent spotlight traction apparatus is probably the actual treatment
3. an incandescent light bulb (nonfrosted) benefit.
4. an electric stove burner element 2. this technique actually does produce
lumbar distraction
157. The purpose(s) of stimulating denervated muscle 3. the pull of the traction is usually adjusted
is (are): B such that it helps eliminate the lumbar lordosis.
1. help lymphatic venous drainage from the 4. the benefit from the traction is strictly
muscle psychological.
2. help retard atrophy to some extent
3. help retard loss of muscle elasticity and 162. Lumbar traction requires:A
the shortening of tendons 1. a distraction force of approximately one
4. help the patient psychologically, in that half the body weight just to overcome the friction
stimulation makes patient feel that between the patient and the table (patient supine on
“something is being done” fixed surface table)
2. a split table in which the upper half of the
158. Some experimental evidence indicates that table is fixed and the tower half is on rollers to
stimulation of denervated muscles is useful in minimize friction between the patient and the table.
preventing atrophy under which of the following 3. a cooperative, patient who will allow the
conditions:C trunk muscles to relax as the distraction force is
1. the patient believes in the treatment applied
2. the stimulation is done several times per 4. a force of least 700 pounds to get actual
day distraction
3. the stimulation is done in the physical
therapy department of a hospital rather 163. the Intervertebral foramina are greatest in size
than at home when.C
4. the contractions of the muscle from 1. the neck is full extension
stimulation are done against external 2. the neck is in full flexion
resistance. 3. traction is given with the neck in
extension
159. When using alternating current stimulation :B 4. traction is given with the neck in flexion
1. the optimum frequency 1 or treating
denervated muscles 164. Which of the following are important guidelines
2. the optimum frequency for treating to the use of cervical traction?:B
denervated muscle is 2,000 Hz 1. intermittent traction of 60 pounds an
3. the optimum frequency for treating distract the cervical spine from 3 to 14 mm
normally innervated muscle is 1,000 to 2,00 Hz 2. the amount of distraction force needed for
4. interrupted galvanic and faradic currents cervical traction is about the same as that
have been shown to retard atrophy of denervated needed for lumbar traction.
muscle to greater degree than alternating currents. 3. distraction of the spine widens the disc
space and spreads the facet joints
160. Lumbar traction:B 4. the increased length of the cervical spine
1. requires at least 200 to 300 pounds of during traction is actually only an apparent
weight to produce any actual skeletal distraction lengthening and is really due to straightening of the
2. used continuously at 10 pounds for 30 normal cervical curve
minutes does distract the lumbar spine
3. distract the vertebrae of the lumbar spine 165. Cervical traction :C
approximately 1-3 mm when at least 300 pounds are 1. should be used such that the pull exerted
used. by the traction halter pulls the neck into
4. doe not distract the spine at any weight hyperextension
used up to 1,000 pounds
2. should be used such that the pull exerted
by the traction halter pulls the neck into flexion
3. has few or no contraindications
4. should be used with caution in patients
with arterial disease and connective tissue disorders.

Chapter 3
Therapeutic Exercise

Directions: For each of the incomplete statements


below, ONE or MORE of the numbered completion is
correct. Each case select:
A. if only 1,2 and 3 are correct
B. if only 1 and 3 are correct
C. if only 2 and 4 are correct
D. if only 4 is correct 66. Body building techniques that commonly used for
E. if all are correct producing muscle hypertrophy: C
1. provide an adequate stimulus for
generalized aerobic conditioning
61. The maximum strength of a muscle is:B 2. usually involve the used of multiple ( 3 7)
1. the maximum tension developed per unit sets of 9 16 repetitions on alternate days
of cross-sectional area 3. often consist of a program of decreasing
2. occurs mainly when the muscle is used in numbers of repetition with increasing resistance.
an isometric manner 4. use maximum resistance (10 RM)
3. most likely to occur when the muscle is at constantly rather than incremental increases or
resting length decreases in resistance load
4. possible only during an eccentric
contraction 67. A power program for weight lifter:A
1. is designed to produce muscle bulk or
62. Maximum strength can be obtained with:E endurance
1. low weight (40% maximal) repetitions if 2. is a program of decreasing numbers of
done to the point of fatigue repetitions with increasing weight until the last
2. high weight (85 to 100% maximal) weight is the greatest that can be moved once.
repetitions if done to the point of fatigue 3. is usually done on five to seven days each
3. isometric muscle strengthening technique week.
if the final measurement of strength is done 4. is good stimulus for aerobic physical
isometrically
4. isotonic muscle strengthening technique if 68. Isometric exercise: E
the final measurement of strength is done isotonically 1. is often clinically in a form called BRIME
2. improves muscle strength without muscle
63. Progressive resistive exercise:E hyperthropy
1. was first described by Delorme 3. does not significantly improve joint range
2. is based on weekly increases in the 10 of motion
repetitive maximum (RM) 4. is often difficult to measure in terms of
3. can be used even if the muscle is not anti the muscle tension being exerted
gravity in strength
4. begins with 10 repetitions of one-half 10 69. Synchronization refers to: D
RM and ends with 10 RM 1. maximal isometric tension
2. minimal isometric tension
64. Exercise to improve strength in muscles is the 3. the pattern of agonist cooperation units to
greatest that can be moved: :E increase strength
1. should be done with caution and careful 4. the coordinated firing of motor units to
monitoring of strength increase strength
2. can be done by placing the joint moved in
a gravity eliminated position 70. Biofeedback training:E
3. can be done with an assistive devise 1. involves the use of equipment that reveals
moved such as a powder board to the patient some internal parameter or event that
4. can be done with a cable and pulley would otherwise have not been consciously known
assistive system such as that used in the Elgin to the patient
exercise table. 2. requires that the patient receiving the feed
back volitionally use it to accomplish a desired goal
65. The Oxford Technique:B 3. using electromyographic equipment often
1. is essentially like the PRE program of improves the results of an exercise program
Delome except that the exercise is arranged from the especially if the desired goals include muscle
heaviest to the lightest weight relaxation or specific muscle firing
2. is an isometric exercise technique 4. is probably the most useful clinically
3. was designed to allow the use of lower
resistance as muscle fatigue ensued 71. Some of the most important parameters in
4. has definitely been shown to provide a evaluating the patient who is going to receive
faster improvement in strength than the PRE neuromuscular facilitation exercise include. E
technique of De Lome
1. the distribution of abnormal tone and spastiolty in 4. superficial
the trunk and extremities.
2. voluntary movements and the effect of abnormal 77. Noxious stimuli. E
tone on them 1. produce a generalized flexion synergy withdrawal
3. coordination of voluntary movements 2. produce the some general effect as tickling
4. motivation of the patient for the treatment and for 3. relax extensor tone by inhibition flexor anatagonist
improvement. 4. are often used as part of a PNF program

72. the peripheral influences on motoneuron 78. Rood’s technique of superficial cooling of the
excitability usually used in propreceptive skin over a muscle or brushing the skin over a muscle
neuromuscular facilitation exercise techniques are. C is. A
1. cerebellar and corticospinal input to the alpha 1. useful in increasing the activity level of a muscle
motor neurons 2. based on the observation that skin stimulation over
2. muscle receptors such as the intrafusal muscle a muscle increases the gamma efferent activity to the
system muscle spindle system.
3. psychological influences such as a emotion 3. due to a multisynaptic reflex connecting skin
4. different input to the alpha motor neurons from the receptor and muscles underlying the skin and which
skin and tendon receptors. in an evolutionary sense may have been useful in
avoiding capture
73. An acceptable facilitatory stimulus should be 4. not explainable with current neurophysiological
minimally capable of causing.: D information
1. motorneuron discharge
2. motorneuron inhibition 79. Central facilitation techniques. B
3. clonus 1. utilize synergies associated reactions and radiation
4. at least sub threshold affect toward discharging a to get facilitator or inhibition of a muscle
motoneuron 2. utilize stroking or icing techniques
3. include the common practice of having a
74. The main difference between the therapeutic hemiplegia patient maximally activate the muscle on
effects of quick and slow passive stretch of a muscle the normal side while to voluntarily use the desired
is. D muscle on the affected side.
1. there is actually no significant therapeutic 4. should not be simultaneously with PNF techniques
difference
2. the pain experienced by the patient is less with Directions: For each of the incomplete statements
slow stretch below, ONE or MORE of the numbered completion is
3. the slow stretch is used only when muscle exhibit correct. Each case select:
clonus A. if only 1,2 and 3 are correct
4. a muscle inhibitory or relaxing effect is obtained B. if only 1 and 3 are correct
by slow stretch, while fast stretch is excitatory and C. if only 2 and 4 are correct
facilitaties muscle activity D. if only 4 is correct
E. if all are correct
75. In classical proprioceptive neuromuscular
facilitation (PNF) techniques, a therapist might 106. Exercise programs for the hand in patients with
stretch a muscle spindle: E inflammatory arthritis are controversial because. :A
1. facilitate voluntary activity in the muscle 1. many authorities feel that any exercise increase the
2. increase the firing rate of the afferents from the loss of joint stability
muscle spindle 2. exercise takes too much of the patient’s time and
3. inhibit voluntary activity in the antagonistic has been proven to be definitely effective
muscle 3. hand exercise programs are very complicated and
4. focus the patient’s attention on the movement of must be designed for each patient and even for each
the muscles finger joint
4. they are clinically proven to be useful following
76. Proprioceptive neuromuscular facilitation (PNF) hand surgery in patients with inflammatory
programs typically include. : A
1. deep cooling of muscle 107. The prescription of exercise for the patient with
2. muscle stretching simple recurrent low back pain should achieve at
3. withdrawal reflexes
least the following. E 1. abdominal muscle strengthening
1. improved low back flexibility 2. reflex inhibition of back musculature
2. improved posture with minimized lumbar lordosis 3. stretching of the low back structures to present
3. improved body mechanism in all activities and fixed excessive lumbar lordosis
exercise 4. permanent reduction of the lumbar lordosis.
4. improved strength of abdominal and hip extensor
muscles 113. Stretching the hip flexors is often useful in
patients with hip, flexion contractures and low back
108. Probably the most useful technique for treating pain. In this type of exercise. B
simple recurrent low back pain is D 1. there is active use of the Thomas test for hip
1. traction flexion contracture
2. pelvic tilt exercise 2. the patient lies supine with both hips in full
3. knee-chest exercise extension
4. bed rest 3. the patient kneels on one knee and rocks back and
forth
109. Prescribing leg lifts in patients with simple 4. the patient lies prone
recurrent low back pain may E
1. exacerbate the [ain as the iliopsoas muscle pulls 114. A hook-lying (or hook-line) sit-up is B
the lumbar spine into lordosis 1. done in the supine position with the knees flexed
2. strengthen the iliopsoas muscle and to some extent 2. used to strengthen the hip flexors
the abdominal muscles 3. used to strengthen the abdominal muscles and to
3. be less helpful than prescribing book-line sit-ups some extent the hip flexors
4. increase intradiscal pressure in the lumbar spine. 4. particularly useful during episodes of acute back
pain
110. knee-chest exercise are used in patients with low 5. particularly useful during episodes of acute back
back pain to E pain
1. strengthen the hip flexors
2. stretch the hamstring muscle 115. Bridging is a useful exercise in patients with low
3. strengthen the hamstring muscles back pain if it can be done without exacerbating the
4. stretch the soft tissue structures of the low back pain because it strengthens the D
1. abdominal muscles
111. hamstring muscle stretching is often done in 2. hip flexor muscles
patient with low back pain :A 3. neck flexor muscles
1. with the patient sitting, knees extended, bouncing 4. back and hip extensor muscles
the trunk forward to attempt to touch the toes.
2. with the patient standing, trying to touch the toes. 116. postural exercise in scoliosis: E
3. with the patient sitting with one knee extended and 1. are used while in the Miwaukee brace since this
one flexed, with truncal bouncing to touch the touch brace forces the patient to extend the spine to prevent
the toes of the extended side. painful pressure on the chin or neck
4. patient with low back pain should be stretching the 2. are used to elongate the spine low back are used to
hamstring muscles since these exercises increase the elongate the spine
lumbar lordosis 3. are used to reduce the lumbar lordosis and dorsal
kyphosis
112. Pelvic tilt exercise is often prescribed for patient 4. can be done with weight on the head to improve
with low back pain because : A the strength of the muscle that distract the spine
1. abdominal muscle strengthening
2. reflex inhibition of back musculature 117. exercise in patients with scoliosis has been
3. stretching of the low back structures to present shown to C
fixed with truncal bouncing to touch the toes of the 1. control and even improve the scoliotic curve
extended side. 2. increase the flexibility of the concave side of the
4. patients with low back pain should not be spine curvature
stretching the hamstring muscles since these exercise 3. worsen the curvature when used without adequate
increase the lumbar lordosis bracing
4. improve respiration
112. Pelvic tilt exercise is often prescribed for
patients with low back pain because A 118. Cotrel’s exercises for scoliosis include E
1. elongation of the spine
2. derolation of the spine 124. Ambulation exercise with immediate
3. lateral flexion of the spine postoperative fit prosthesis should C
4. self powered cervical traction (the lower 1. involve no weigh bearing on the prosthetic side of
extremities push on a bar to provide the power for six weeks
cervical traction through a double pulley system) 2. involve only touchdown weight bearing the first
day after amputation
119. Buerger’s exercises for treatment of peripheral 3. achieve full weight bearing by the fourth
vascular disease E postoperative week
1. have not been shown to be effective in controlled 4. involve progressively more weight bearing on the
2. do not increase muscle blood flow temporary prosthesis until it is bearing a maximum of
3. are based on the theory that alternately distending 15 pounds (6 kg)
and emptying blood vessels would make them
capable of handling greater quantities of blood 125. The most appropriate exercise regimen for a
4. originally included the use of an electric heating patient with below-knee amputation and immediate
pad or hot water bottle on the limbs. postoperative fitting is to B
120. Buerger-Allen exercise A 1. begin with touchdown weight bearing on the
1. Combine the postural program of Buerger with prosthesis and progress to a three-point gait using
active plantar flexion and dorsiflexion of the ankle at parallel bars, crutches, or a walker
rate of 30 min 2. begin with three-point gait using axillary crutches
2. are less effective in increasing blood flow to 3. after fitting the permanent prosthesis, focus on
muscles than resistive exercise of the lower obtaining equal stride length and elimination of
extremities prosthesis gait deficiencies
3. increase the quantity of blood flowing to lower 4. begin with a four-point gait using axillary crutches.
extremity muscles
4. include walking to the point of claudictory pain 126. The exercise program for a lower extremity
amputee not fitted with an immediate post operative
121. Achieve muscle contraction; B prosthesis is aimed at: E
1. reduces the quantity of blood flowing in a muscle 1. preventing contractures of the hip and knee
during the contraction 2. preventing loss of strength of the hip extensors,
2. are less effective in increasing blood flow to flexors, and abductors as well as the knee extensors
muscles than resistive exercise of the lower and flexors
extremities 3. rapid shrinkage of the stump (elimination of post
3. increase the quantity of blood flowing in a muscle operative edema)
between contractions 4. preservation of the strength and range of motion of
4. decreases the quantity of blood flowing in a muscle joints in the remaining extremities
between contractions.
127. The treatment program for an infant with
122. The most effective treatment (nonsurgical) of the acquired flat feet who is not old enough to walk
following treatment programs for peripheral vascular should include E
disease is:D 1. passive external rotation of the hips by the parents
1. Buerger’s exercise 2. passive internal rotation of the hips by the parents
2. vasodilators 3. massaging of the feet at least four times per day
3. Buerger-Allen exercise 5. passive foot stretching
4. brisk walking to the point of claudicatory pain (for by the parents in which
at least one hour per day) the forefoot is adducted
while the heel is
123. The main advantages of immediate inverted
postoperative fitting of a lower extremity prosthesis
are E 128. Treatment program for an infant with acquired
1. psychological benefits to the patient of early flatfeet who is not old enough to walk should
ambulation include:C
2. early re establishment of prioprioceptive feedback 1 passive external rotation of the hips by the parents
by contact with floor 2. passive internal rotation of the hips by the parents
3. pain relief by prevention of stump edema 3.massaging of the feet at least four times per day
4. prevention of complications of prolonged bed rest
4.passive foot stretching by the parents in which the 135. In treating gait dysfunction it is important to
forefoot is adducted while the heel is inverted bear in mind that gait is affected by E.
1. a person’s body image
129. Treatment of acquired flat feet in a child old 2. neurological muscle control competency
enough to walk might include: E 3. conscious and unconscious sensory feedback
1. shoes having an adducted forefoot pattern 4. perceptual orientation competence
2. a lift under the medial side of the heel
3. attaching a bar to the sole of the shoes at night to 136. The technical difference between walking and
hold the hip in internal rotation running exercise is that the D
4. stretching the Achilles tendons 1. stance phase of gait is decreased in running
2. swing phase of gait is increased in running
130. The treatment of the older child with flat feet 3. period of double stance (double support) is longer
usually includes: B in walking
1. stretching the Achilles tendons 4. period of double stance (double support) occurs
2. stretching the foot evertors only in walking
3. strengthening the foot inventors
4. encouraging the child to walk with feet pointing 137. Gait speed is :A
outward 1. unconsciously chosen by each individual at the
speed giving the lowest energy
131. Treatment for foot strain in the adult might 2. about 3 mph for adult males
include: A 3. unconsciously chosen by each individual such that
1. toe pick-up exercises the pendulum periodicity of the lower extremities fits
2. standing with the feet in slight inversion the cadance
3. weight loss down to ideal weight 4. only 3mph for adult males because the slower you
4. one-eight inch wedge in the lateral border of the walk the less energy required for ambulating any unit
heel of the shoe distance

132. Treatment program for a child with pigeon toes 138. Gait speed is unconsciously chosen by each
must be based on the type of lower extremity individual to allow maximum use of: C
pathology producing it, such as A 1. large rather than small muscles
1. medial hip rotation, usually associated with 2. gravity and inertia rather muscles
2. a history of “W” sitting 3. concentric rather than eccentric contractions of
3. metatarsus varus muscles
4. external hip rotation, usually associated with a 4. eccentric or lengthening contraction of muscles
history of lying prone with lower extremities in the
frog leg position 139. Loss of motion at which joint produces the least
increases in the energy required for ambulation D
133. The typical exercise program for pigeon toeing 1. knee
due to internal tibial torsion includes. B 2. hip
1. passive rotation of the tibia and fibula into external 3. both hips
rotation 4. ankle
2. passive external rotation of the femur
3. “strengthening” the leg by grasping the medial 140. A gait characterized by reduction of stance phase
aspect of the knee and ankle and using the thumbs to on one side with sudden movements downward of the
push against the fibula head, arms and trunk at stance phase on the same side
4. a shoe bar can be used to keep the femur on each is D
side in internal rotation during the night 1. due to gluteus medius weakness
2. due to anterior tibial muscle weakness
134. Pigeon-toed due to metatarsus varus is best 3. hysterical
treated by D 4. antalgic
1. casting of the foot in every case
2. encouraging “w” sitting 141. Some of the gait characterized of normal aging
3. using cable “twister” include E
4. passively stretching the forefoot toward normal 1. slower walking speed
alignment 2. shorter stride length
3. wider base of gait
4. relatively more time is spent in distance phase

142. Gait in Women is B


1. slower than in men (mainly due to shorter stride
length rather to slower cadenced
2. not related to the rule of choosing gait speed based
on the least energy expenditure per unit distance
3. slower than normal in high heels
4. faster than men in a crowded area

143. The normal aging process produces an


involuntional gait involving a flexed attitude, dorsal
kyphosis, mild proximal muscle rigidity, decreased
automatic movements, and a decrease in the
amplitude of associated movements, tending to
indicate involutional changes D
1. pyramidal system
2. skeletal muscle
3. autonomic nervous system
4. extrapyramidal system

144. A prescription of progressive ambulation might


include E
1. tilt table therapy
2. pool walking
3. walking in the parallel bars
4. walking with asistive devices such as crutches,
cane, walker, etc.

145. In very severe gait, problems the wheelchair


may :E.
1. be a liberating rather than confining device
2. be better form of ambulation because of the much
lower level of energy required to travel per unit of
distance
3. be safer for some patients than attempts at walking
4. have to be self-powdered (usually electric) if the
upper extremities are also impaired.

Chapter 4
Neurophysiology
Directions: For each of the incomplete statements
below, ONE or MORE of the numbered completion is
correct. Each case select:
A. if only 1,2 and 3 are correct
B. if only 1 and 3 are correct
C. if only 2 and 4 are correct
D. if only 4 is correct 6. During voluntary muscle contraction (light to
E. if all are correct maximal effort) the discharge rates of single
motoneurons varies between D
1. What is the resting transmembrane potential in A. one to five impulses per second
muscle? C B. one to 20 impulses per second
A. – 70 mV C. five to 20 impulses per second
B. + 70 mV D. five to 50 impulses per second
C. – 90 mV E. one to 60 impulses per second
D. + 90 mV
E. there’s none 7. What is the most conspicuous sign of spinal shock
in man and other primates? E
2. What is the “resting transmembrane potential in A. loss of motor strength
nerve? A B. loss of sensation
A. –70 mV C. presence of autonomic hypereflexia
B. +70 mV D. hypereflexia below the level of cord transection
C. –90 mV E. areflexia or hypereflexia below the level of cord
D. +90 mV transection
E. there is none
8. What are the first reflexes to emerge from the
3. Which of the following is the correct sequence period of areflexia following spinal cord transection? C
when considering the production of an action potential A. tendon jerks
D B. anal reflexes
A. increase in sodium permeability; increase in C. withdrawal movements in response to plantar
potassium permeability; decrease in sodium and stimulation
potassium permeabilities simultaneously; active D. genital reflexes
transport of sodium out of nerve fiber E. none of the above
B. increase in sodium permeability; increase in
potassium permeability; decrease in potassium 9. The higher centers of the nervous system control a
permeability while sodium permeability remains spinal reflex by exerting their influence on which of the
constant; decrease in sodium permeability; diffusion following? D
of sodium out of the nerve and potassium into the A. motoneurons
nerve B. intemuncial neurons of the reflex arc
C. simultaneous increase of sodium and potassium C. gamma motoneurons, which set the sensitivity of
permeabilities; simultaneous decrease of sodium and the stretch afferent organs
potassium permeabilities; simultaneous diffusion of D. all of the above
sodium out of and potassium into the cell E. none of the above
D. increase of sodium permeability; increase of
potassium permeability with simultaneous decrease of 10. Body temperature is regulated to what part of the
sodium permeability, decrease of potassium central nervous system? E
permeability, active transport of sodium out of nerve A. right parietal lobe
fiber, with concurrent active transport and diffusion of B. pons
potassium into nerve above. C. limbic system
E. none of the above D. thalamus
E. hypothalmus
4. Saltatory conduction in nerves is a result of which
one of the following? C 11. What are the efferent cells of the cerebellar
A. myelination of only large diameter axons cortex? B
B. myelination of only small diameter axons A. Betz’z cells
C. myelination of any size nerve B. Purkinje’s cells
D. loss of myelination of very large diameter nerve C. Cuboid cells
E. changing the potassium ion concentration in the D. Schwann’s cells
intestinal fluid about a nerve E. none of the above

5. What is the name applied to what happens to the 12. Increasing degrees of heat first stimulate warmth
distal portion of an axon when the axon is severed? D endings and then commence to stimulate pain
A. neuropraxia endings. At what temperature are pain endings first
B. axonotmesis stimulated? D
C. neurotmesis A. 16.4 C
D. wallerian degeneration B. 23.7 C
E. none of the above C. 25.2 C
D. 44.9 C
E. 59.9 C
1. III
13. The Weber and Rinne diagnostic test are used to 2. VII
test which sense? D 3. IX
A. vision 4. XI
B. taste
C. touch 20. The autonomic nervous system innervates:A
D. hearing 1. smooth muscle
E. smell 2. cardiac muscle
3. glands
14. Broca’s area is typically associated with which 4. skeletal muscle
term? A
A. expressive aphasia 21. damage to the basal ganglia can result in which
B. receptive aphasia disorders?: E
C. apraxia 1. athetosis
D. auditory agnosia 2. chorea
E. visual agnosia 3. ballismus
4. Parkinson’s disease
15. Which hormone is concerned with water
regulation? C 22. Dysmetria is a deficiency ion which of the
A. prolactin following?C
B. oxytocin 1. force
C. ADH (antidiuretic hormone) 2. rate
D. Epinephrine 3. steadiness
E. nonrepinephrine 4. direction

Directions: For each of the incomplete statements 23. With reference to sensory pathways of the spinal
below, ONE or MORE of the numbered cord, the anterolateral tracts exclusively conduct
completions is correct. In each case select. impulses concerning which kind(s) of sensation?:E
A. if only 1,2 and 3 are correct 1. pain from skin, muscles, tendons joints and viscera
B. if only 1 and 3 are correct 2. warmth
C. if only 2 and 4 are correct 3.cold
D. if only 4 is correct sexual sensations
E. if all are correct
24. With reference to sensory pathways of the spinal
16. When considering muscle contraction calcium cord, the posterior white colums conduct impulses
ions. E concerning which kind(s) of sensation?: E
1. stimulate the Atpase of aytomyosin 1. proprioception
2. are concerned mainly with control of muscle 2. touch-pressure
contraction 3. kinesthesia
3. stimulate the ATPase of myosin 4. those related to perceptual functions
4. play a more important physiologic role than
magnesium ions 25. Irritaion of tyeh viscera by a pathologic process is
manifested by: E
17. The sympathetic division of the autonomic 1. pain
nervous system originates from pregganglionic 2. somatic reflexes, muscular rigidity
neurons in C 3. autonomic reflexes – sweating, piloerection, or
1. certain cranila nerve nuclei vasomotor changes
2. the thoracic spinal segments 4. hyperalgesia, hyperesthesia, or tenderness
3. the second third, and fourth sacral segments of the
spinal cord 26. Which cranial nerves are concerned with
4. the upper lumbar spinal segment sensation of the tongue? A
1. VII
18. The stellate ganglion: c 2. V
1.is formed by the fusion of the superior and middle 3. IX
cervical ganglia 4. X
2.is formed by the fusion of the inferior and first
thoracic ganglia 27. Which of the following are forms of agnosia : A
3. is part of the parasympathetic nervous system 1. failure to recognize common objects by palpating
4. is part of the sympathetic nervous system them
2. inability to appreciate the meaning of objects seen
19. Which cranial nerves are involved in the of colors or of visual space in the absence of a
parasympathetic nervous system?: A primary visual defects.
3. failure to recognize the parts of the body, to
differentiate right and left or in general to recognize
the relationship of objects to the body.
4. inability to perform voluntary movements in the
absence of motor paralysis.

28. Water deprivation is the usual physiologic


situation that elicit drinking behavior. Which of the
following are neural mechanism responsible for this
drinking behavior? C
1. thalamic osmoreceptors
2. hypothalamic osmoreceptors
3. osmoreceptors in the limbic system
4. cells of the lateral hypothalmus

29. Which of the following are properties of axonal


transmission of impulses:B
1. conduction in both directions
2. greater susceptibility to asphylaxia, ischemia, and Chapter 5
depressant drugs than synaptic transmission of Eletromyography
impulses
3. evovation of one action potential by each stimulus Direction: Each of the question or incomplete
(disregarding refractory period) statements below is followed by five suggested
4. unidirectional conduction
answer or completion. Select the single BEST answer
30. The gamma efferent system is excited primarily by in each case.
the D
1. cerebellum 1. Electrodiagnosis is useful in detecting
2. cerebral cortex abnormalities of E
3. basal ganglia A. muscle fibers
4. bulboreticular facilitatory region of the brain stem. B. peripheral nerves
C. myneural junctions
D. anterior horn cell
E. all of the above

2. The “motor unit” consists of all of the following


except the A
A. cerebral cortical motor strip
B. anterior horn cells
C. axon
D. myoneural junction
E. muscle

3. What is the proper definition of term “motor


point”? D
A. the point along a peripheral nerve where motor
fiber only are found
B. the ventral root of spinal nerve consists only of
motor nerve fiber and is referred to as the motor
point.
C. the anterior horn cell is the “motor point” of the
motor unit, and the term refers to the location of
anterior horn cells in the spinal cord.
D. the “motor point” refers to the point where the
motor nerve enters the muscle
E. none of the above

4. The term “innervation ratio” refers to the C


A. number of individual nerve root which innervate a
specific muscle 10. When a muscle is voluntarily contracted, the
B. number of muscle fiber peripheral nerves which motor units will begin to fire. What is the usual initial
innervate a specific muscle rate of firing of motor unit during a week contraction.
C. number of muscle fiber per motor nerve fiber A. 1 to 3/min
D. number of nerve contained in the peripheral nerve B. 5 to 15/min
E. none of the above C. 5 to 15/sec
D. 75 to 100/sec
5. Nerve conduction velocity is most dependent upon. E. none of the above
C
A. length of entire nerve 11. In a patient complaining of weakness, the
B. size of the muscle electromyogram reveals polyphasic motor unit action
C. diameter of the nerve fiber potentials. These polyphasic potentials account for
D. intensity of the stimulus about 10% of motor unit action potentials studied.
E. none of the above The remainder of the study is normal. How should
these findings be interpreted? E
6. In Lloyd’s Roman numeral classification system of A. suggestive of myopathy
sensory nerve fibers. Group of fiber will have the B. diagnostic of myopathy
highest conduction velocities? A C. suggestive of a neuropathic process
A. group I D. diagnostic of neuropathic process
B. group II E. none of the above
C. group III
D. group VI 12. In routine nerve conduction studies what would
E. none of the above, since Lloyd system does not be the approximately normal value obtained for the
categories fiber either by their size or conduction velocity of nerve conduction? D
speed A. 0.5 m/sec
B. 5m/sec
7. The term “volume conduction” refers to the D C. 15 m/sec
A. absolute frequency range of the electromyography D. 50 m/sec
(EMG) machine speakers E. 100 m/sec
B. ability of EMG machine amplifier to handle large
amplitude electrical discharges 13. A patient presents with the following nerve
C. conduction along a peripheral nerve involving conduction values:
both sensory and motor fibers ulnar nerve motor latency = 3.2 m/sec
D. spread of current through a conducting medium ulnar nerve sensory latency = 3.0 m/sec
from its source median nerve motor latency = 4.0 m/sec
E. none of the above. median nerve sensory latency = 4.2 m.sec
median nerve motor conduction velocity = 57 m/sec
8. In normal muscle at rest which following potential what is a likely diagnosis in this case? C
is common fibers? E A. ulnar neuropathy at the elbow
A. fibrillation B. ulnar nerve entrapment at the wrist
B. positive wave C. carpal tunnel syndrome
C. “myopathic” motor unit action potential D. peripheral polyneuropathy
D. myotonic discharge E. these are the normal findings
E. none of the above
14. A patient presents with what appears to be a
9. Following insertion of the needle recording pronator syndrome. Which nerve is compressed in
electrode into a muscle, a patient complain of pain this syndrome? E
the EMG scope shows low amplitude and short A. tibial
duration potentials which have an initial negative B. peroneal
deflection. These potentials most likely represent E C. ulnar
A. positive waves D. radia;
B. fibrillation E. median
C. myotonic discharges
D. bizarre high frequency potentials
E. end-plate potentials
15. The most common site ulnar nerve compression is D. Flexor hallucis longus, abductor hallucis and
D tibialis anterior
A. in the palm E. extensor digitorum brevis, peroneus longus and
B. in the palm tibialis anterior
C. at the wrist
D. in the forearm 19. A patient was stabbed with a knife and the ulnar
E. at the elbow nerve was severed at the elbow. Nerve conduction
F. at the thoracic outlet studies are performed with the following results:
1. no response in the abductor digiti quinti when the
16. A patient complains of hyperthesia in the ulnar ulnar nerve is stimulated
nerve distribution. Nerve conduction studies are 2. an evoked response with a latency of 3.8 m/sec is
performed and reveal the following results: obtained from the abductor digiti quinti when the
median nerve motor distal latency =3.6m/sec ulnar nerve is stimulated at the wrist.
median nerve motor conduction velocity in the
forearm==55 m/sec. What is a likely explanation for these results? D
Ulnar nerve motor distal latency = 3.2 m/sec A. the ulnar was not totally severed but is still
Ulnar nerve motor conduction velocity across in the partially functioning
forearm = 52 m/sec B. the response recorded from the stimulation at the
Ulnar nerve motor conduction velocity in the arm = wrist is a volume conducted response from the
62 m/sec median innervated hand muscles
What is the likely diagnosis? D C. there is spread of the stimulus to the median nerve
A. carpal tunnel syndrome when the nerve is stimulated at the wrist.
B. pronator syndrome D. there is median-ulnar crossover
C. thoracic outlet syndrome E. the studies are definitely in error and should be
D. ulnar nerve compression at the elbow repeated
E. insufficient data are given for any conclusions
20. A patient has the following EMG findings:
17. Femoral nerve are performed on a patient the 1. positive waves and fibrillations in the proximal
following results are obtained: girdles muscles
latency from above the inguinal ligament to the 2. low amplitude, short duration motor unit action
vastus medials muscle = 9.7 m/sec potentials
latency from below the inguinal ligament to the 3. normal nerve conduction and respecyive
vastus from below medialis muscle = 5.8 m/sec stimulation studies
distance between the two sites of stimulation = 12cm what would be the most likely diagnosis? C
what would be likely diagnosis?D A. myasthemia gravis
A. the studies are normal B. myotonic dystrophy
B. diabetic amyotrophy C. polymyositis
C. peripheral neuropathy D. amytrophic lateral sclerosis
D. femoral nerve compression at the inguinal E. G ilain-Barre syndrome
ligament
E. femoral nerve injury at level of the psoas muscle 21. An EMG performed during the period of flaccid
paralysis in the disease known as hypokalemic
18. After a prolonged period of sitting with the legs periodic paralysis reveals. E
crossed, a patient complain of dragging the right foot. A. normal findings
The patient is able to walk on her toes but cannot B. positive waves and fibrillations
walk on the heel of her right foot due to weakness. C. bizarre high frequency potentials
An EMG examination is planned. D. long duration motor unit action potentials
Which muscle would be expected to show EMG E. electrical silence
abnormalities? E
A. extensor digitorum brevis, tibialis posterior and 22. A patient presents with symptoms typical of an
extensor hallucis longus acute radiculopathy. The EMG examination is normal
B. tibialis anterior, tibialis posterior, and peroneus but the H reflex response is absent on the affected
brevis side. A normal H reflex response is obtained on the
C. extensors hallucis longus, abductor hallucis and uninvolved side. Which nerve root is probably
peroneus tertius compressed? E
A. C6
B. C7
C.C8
D. L5
E. S1

23. A patient has a right facial palsy (Bell’s palsy).


Nerve conduction studies are performed and revel a
latency of 3.4 m/sec to the frontalis muscles and 3.3
m/sec to the orbicularis oris muscle. The facial palsy
ahs been present for 24 hours prior to the above
studies. What can be said about the prognosis? E
A. excellent for full and rapid recovery
B. expected full recovered but over a period of about
six weeks
C. expected incomplete recovery
D. poor, with little recovery
E. none of the above
Chapter 6
Orthotics and Ptosthetics

Directions: For each of the incomplete statements


below, ONE or MORE of the numbered
completions is correct. In each case select.
A. if only 1,2 and 3 are correct
B. if only 1 and 3 are correct
C. if only 2 and 4 are correct
D. if only 4 is correct
E. if all are correct
1. A mechanical object that support a body part is
called a (n) E
1. brace
2. othosis
3. orthesis
4. splint

2. Some of the basic reasons of a deformity of a body


device include A
1. support of body part
2. immobilization of a body part
3. prevention or correction of a deformity of a body
part
4. replacement of a body part

3. Some of the characteristic of a successful orthosis


include E
1. durability
2. comfort and cosmetic appearance
3. lightness in weight
4. strength

4. An orthotic prescription should include E


1. the body part to be braces
2. the type of brace
3. the type of material
4. the type of joint and other special considerations
5. The most similar devices from a prosthetic and 1. a long opponens orthosis
orthotic standpoint are D 2. a short oponens orthosis
1. lower extremity prosthetic and orthotics 3. a spring –loaded ankle joint
2. upper extremity orthotics and lower extremity 4. a flexor hinge splint
3. lower extremity orthotics and upper extremity 79. The level of quadriplegia (lowest spared level)
4. upper extremity orthotics and prosthetics most likely to benefit from a flexor hinge splint is D
1. C-4
6. Some of the reasons a patient might reject 2. C-8
anorthosis include E 3. C-7
1. no useful activity in which the patient could use in 4. C-6
device 80. The level(s) of quadriplegia most likely to benefit
2. difficulty in putting the orthosis on and taking it off from long opponents orthosis is fare. C
3. negative attitudes of the prescribing physician and 1. C-6
other rehabilitation team members 2. C-5
4. cosmetic appearance 3. C-7
4. C-4
7. Leather is till in orthotics because it has some Chapter 7
characteristics that are unmatched by any synthetic Spinal Cord Injuries
material, including A
1. water vapor permeability Direction: Each of the questions or incomplete
2. ability to absorb water from moist air surrounding statements below is followed by five suggested
the skin answer or completions. Select the single BEST
3. ability to absorb ,and disperse moisture from points answer in each case.
to heavy sweating
4. ability to retain its shape 1. The approximate number of persons with spinal
cord injury and paraplegia or quadriplegia currently
Directions: For each of the incomplete statements residing in the United States is about D
below, ONE or MORE of the numbered A. 5,000
completions is correct. In each case select. B. 10,000
A. if only 1,2 and 3 are correct C. 100,000
B. if only 1 and 3 are correct D. 200,000
C. if only 2 and 4 are correct E. 800,000
D. if only 4 is correct
E. if all are correct 2. Cervical spine immobilization is desired during
74. The principal use of lower extremity orthotic healing of a cervical fracture. In case where surgical
devices include E stabilization is not planned, how might adequate
1. support of body water immobilization best be achieved? D
2. prevention of lower extremity deformities A. bedrest with the use of sandbags beside the head
3. correction of lower extremity deformities B. use of a body cast extending up over the mandible
4. control of involuntary movements C. soft collar
D. use of traction through skull tongs
75. Pivot or suspension feeders are prescribed to B E. Philadelphia collar
1. support the thumb in abduction to allow opposition
with the fibers 3. The muscle stretch reflexes during the period of
2. hold the interphalangeal spinal shock are A
joints in full A. absent
3. support the metacarpal arch B. diminished
4. prevent ulnar deviation of the fifth digit C. normal
76. A long opponents orthosis is prescribed to A D. increased
1. support the thumb in abduction to allow opposition E. can be quite variable
with the fingers
2. support the metacarpal arch 4. The term spinal shock refers to:
3. stabilize the wrist A. the hypoexcitability of the portion of the spinal
4. allow flexion of the wrist only cord below the level of a spinal cord injury
78. The reciprocal-wrist extension- finger flexion B. the circulatory shock that occurs following spinal
orthosis is also called D cord injury
C. a transient phenomenon 9. A paraplegia patient sustains a fracture of the
D. all of the above femur during stretching exercises. What is the
probable underlying reason for this fracture? A
5. A patient with C6 quadriplegia is examined by A. osteoporosis
means of electromyography and at the level of the B. osteomalacia
lesion, abnormalities consistent with lower motor C. muscle paralysis
neuron disorder are found. How would this be D. muscle contracture
explained. E. ligamentous laxity

A. any injury to the spinal cord produces lower motor 10. A quadriplegic patient is on respirator. What
neuron findings at below the level of the lesion guidelines might be used to help indicate that the
B. the electromyogram is probably in error since only patient may be able to be taken off the patient may be
upper motor neuron abnormalities should be present able to be taken off the respirator? C
C. the changes seen on the electromyogram reflect A. generally patient can be safely weaned from the
damage to the anterior horn cells and anterior roots respirator at 6 months postonset
D. both A and C B. motor unit action potentials in the stemomastoid
E. all of the above. muscle on EMG examination indicate that the patient
can be removed from respirator
6. Particular ossification has just developed around a C. a vital capcity of 500 to 800 ml is a good
hip and limits hip joint motion. The most appropriate guidelines for the beginning of removal of the patient
treatment B from the respirator
A. immediate surgical removal of the bone D. the patient can be taken off the respirator if he is
B. delayed surgical removal of the bone afebrile and has a normal ECG
C. radiation to the bone E. the patient should be weaned only following a
D. methotrexate laminectomy and fusion
E. disodium edentate
11. The major area of concern is managing the C3
6. Paraarticular ossification has just developed quadriplegic patient is E
around a hip joint and limits hip joint motion. The A. maintaining proper nutrition
most appropriate treatment is B B. preventing pressure ulcers
A. immediate surgical removal of the bone C. preventing contractures
B. radiation to the bone D. strengthening the tongue and muscles of
C. methotrexatre mastication
D. disodioum edentate E. maintaining adequate respiration

7. A patient with quadriplegia develops swelling in 12. A patient with early flaccid quadriplegia develops
the upper thigh. An x-ray of this area reveals what swelling of the leg and low grade fever. What is the
appear to be cloudy, streaky densities in the muscles. likely diagnosis? D
What is the likely diagnosis? A A. urinary tract infection
A. heterotropic ossification B. Undiagnosed fracture of the femur
B. cellulites C. undiagnosed fracture of the tibia
C. thrombophlebitis D. thrombophlebitis
D. acute rupture of the quadriceps mechanism E. arterial embolism
E. Osteogenic sarcoma
13. Immediately following an automobile accident, a
8. The most important way to help combat patient is rendered a C7 quadriplegic. Over the next
osteoporosis in the spinal cord patient is to D two days, the patient seems to develop further
A. administer high doses of vitamin D weakness, and a through examination reveals the
B. administer vitamin D and calcium supplement presence of a C6 quadriplegic. What should be done
C. administer calcitonin in most such cases? A
D. mobilize the patient as early as possible A. nothing
E. perform range of motion exercises passively three B. immediately laminectomy to decompress the cord
times daily C. immediately laminectomy with spinal fusion
D. anterior interbody fusion without laminectomy
E. intrathecal administration of factor VIII to obtain
hemostasis
14. Which of the following statements is most 19. Blood supply to the spinal cord is not equal at all
correct? E spinal segments, some areas better vascularized than
A. early laminectomy is indicated in all cases of others. Which segment is most poorly vascularized?
spinal cord injury C
B. early laminectomy is indicated only in cases of A. C3
quadriplegia B. C8
C. early laminectomy is indicated only in case of C. T4
paraplegia D. T9
D. early laminectomy is indicated only in cases of E. L3
complete cord lesion
20. A quadriplegic patient becomes hypotensive when
15. At the time of laminectomy is usually not raised to the upright position on the tilt table. Why is
indicated a spinal cord injury, the cord appears intact this? A
with no apparent has a complete flaccid paraplegic. A. blood pools in the abdomen and lower extremities
What can be said about the prognosis? E
A. the lesion is probably a neuropraxia and the B. the heart is unable to increase its rate when the
patient should go on to complete recovery in from patient is upright
one to two weeks C. pulmonary ventilation is compromised by the
B. full recovery will occur but will probably occur straps on the tilt table restricting chest expansion
over a one year period D. only A and C
C. the patient will probably be permanently left with E. all of the above
a neurologic bladder but the legs will recover fully.
D. the patient will probably have only some 21. A typical cystometrogram in a patient with a
permanent weakness in the muscles around the feet complete cauda equina injury would look loke D
E. none of the above A. a line with multiple large spikes
B. a line with one large spike
16. a patient sustains a sever fracture of the L1 C. a line with multiple small spikes
vertebral body and is left with neurological deficit. D. a line that is nearly horizontal
What type of deficit would be expected? D E. a line with a rather large upward slope
A. spastic paraplegia with spastic
bladder 22. A potential serious side effect of autonomic
B. spastic paraplegia with flaccid hyperreflexia is E
bladder A. myocardial infraction
C. flaccid paraplegia with spastic B. renal failure
bladder C. Dehydration
D. flaccid paraplegia with flaccid D. Gastric ulceration
bladder E. cerebral hemorrhage
E. paraplegia with no bladder
involvement 23. The apparent mechanism of autonomic
hyperreflexia is A
17. The most frequent type to injury to the spinal cord A. massive release of catecholamines
is C B. inactivation of acytylcholinesterase
A. stretch C. potentiation of the effect of acetylcholine by
B. laceration increasing its duration of release
C. confusion D. inhibition of the vagal responses
D. transection E. central release of dopamine in the striatum
E. all of the above are about equal in frequency
24. In lesions of the cervical spinal cord, bladder
18. The artery of Adamkiewiez is a major feeder this distention may be associated with the development of
artery most commonly found? D E
A. C1 and C4 A. systolic hypertension
B. C7 and T1 B. bradycardia
C. T2 and T5 C. sweating above the level of the lesion
D. T9 and L1 D. cardiac arrhythmia
E. L4 and S1 E. all of the above
E. all of the above
25. In an autonomic reflex type of bladder that is
being treated with intermittent catheterization what is 31. The major cause of death in the paraplegic is C
the acceptable quality of residual urine in the bladder A. pneumonia
A. 5 to 10% of bladder volume B. myocardial infraction
B. 25 to 30% of bladder volume C. renal disease
C. 45 to 50 of bladder volume D. suicide
D. 65 to 70% of bladder volume E. cancer
E. 85 to 90% of bladder volume
32. A patient with acute paraplegia is seen in the
26. Following cervical spinal cord injury, what is the emergency room one hour after the injury. How
status of the bladder after the patient has passed should his bladder be managed in the first several
through the stage shock? B hours following injury? (no renal damage occurred
A. normal function from the injury.) B
B. spastic uninhibited function A. intermittent catheterization should be started with
C. flaccid and areflexic the initial catheterization being performed as soon as
D. ataxic contractions of the detrussor muscle possible
E. the patient often remains anuric B. intermittent catheterization should be started but
the initial catheterization need not be performed until
27. What is the status of the bladder during the period 12-24 hr postinujury
of spinal shock? C C. a Foley catheter should be inserted and left in
A. normal function place to monitor urine output during the immediate
B. spastic uninhibited function postinjury period. Fluids should be forced.
C. flaccid areflexic D. a Foley catheter should be inserted and left in
D. ataxic contractions of the destrussor muscle place to monitor urine output during the immediate
E. the patient is anuric during spinal shock postinjury period. Fluids should be restricted
E. an immediate suprapubic cystostomy should be
28. The term “neurologic bladder refers to perfomed.
A. dysfunction of the bladder owing to peripheral
nerve disorder 33. The most common suprapubic cyctostomy is D
B. dysfunction of the bladder owing to disorders of A. bladder stones
the bladder musculature B. renal stones
C. dysfunction of the bladder owing to central C. peno-scrotal abscess
nervous systems disorders D. infection
D. only A and C E. bladder rupture
E. all of the above
34. The procedure of management of the
29. Following cervical spinal cord injury, what neurologenic bladder in which the bladder is
happened to the sweating pattern? A alaternatly filled and emptied of irrigating fluid is
A. sweating decreases below the level of the lesion called A
B. sweating increases below the level of the lesion A. tidal drainage
C. sweating decreases over the entire body B. intermittent catheterization
D. sweating increases over the entire body C. bladder reeducation
D. a cutaneous vesicostomy
30. A patient who has been quadriplegic for one E. none of the above
month is noted to have a depressed body temperature
after coming in form the cold. Why is this? B 35. Complication from the use of the indwelling
A. it is normal finding for all humans urethral catheter include E
B. the mechanism controlling heat production and A. urinary infection
loss do not properly function in the quadriplegia B. bladder stones
patient C. hydronephrosis
C. the quadriplegia patient has diminished sensation D. peno-scrotal abscess, diverticulum or fistula
and most likely dressed insufficiently for the cold E. all of the above
D. postural hypotension decreased the blood flow to
such an extent that the temperature could not be 36. The term “cutaneous vesicostomy” refers to B
maintained
A. a procedure in which the bowel is directly E. none of the above
connected to the bladder
B. a procedure in which the bladder is directly so that 43. How soon following cervical cord injury would it
they drain into bowel be expected that peristalsic sound might be first
C. a procedure in which the uterus are diverted so heard? A
that they drain into bowel A. within two or three days
D. suprapubic cystostomy with the use of a catheter B. between one and three weeks
E. none of the above C. between three and five weeks
D. between five and nine weeks
37. In a patient requiring the use of an indwelling E. only after 10 weeks
urtheral catheter, what can be done to decrease the
incidence of peno-scrotal fistulae? E 44. What is generally the best time of day for the
A. administer prophylactic antibiotics quadriplegia patient to perform his bowel program? B
B. encourage a high fluid intake A. in the morning just after awakening
C. restrict oral fluids B. after breakfsat
D. tape the catheter to the upper medial thigh C. around noontime
E. tape the catheter to the abdomen D. before the evening meal
E. just before going to bed
38. The procedure of choice for the treatment of
vesico-uretheral reflux is A 45. In patient with a complete spinal cord lesion, the
A. uretheral catheter drainage penis is noted to be enlarged and semi-erected during
B. transurethreal resection (TUR) of the bladder neck the stage of spinal shock. Why? C
C. contraction vesicostomy A. this a reflex erection owing to the stimulation
D. cutaneous vesicostomy provided by the catheter
B. this is probably a psychological erection
39. A useful trechnique or procedure for C. the corpora cavemosa are passively engorged of
demonstrating vesico-uretheral reflux is the E vasodilation
A. cystometrogram D. this represents priaprism which commonly occurs
B. intravenous pyelogram (IVP) with complete cord lesions
C. high infusion IVP E. none of these
D. nephrotomogram
E. cystogram 46. A patient with paraplegia is noted of have a small
ulcer over the lateral malleolus. The likely cause is D
40. The major cause of vesico- uretheral in the A. cellulitis
paraplegic is B. arteriosclerosis
A. anatomical anomaly of the vesico-ureteral junction C. thrombophlebitis
B. uretheral stones D. pressure
C. bladder infection E. diminished skin touhgness
D. ascending infection of the urinary tract
E. peno-scrotal fictula 47. The patient with paraplegia should be instructed
in appropriate therapeutic activities for home use
41. Management of the neurologenic bowel in the after discharge from the rehabilitation unit. Which of
first several days following injury should consist of B the following should not be used by the paraplegia
A. digital rectal stimulation patient? D
B. digital evacuation of feces A. strengthening exercises of the upper extremities
C. use of oral laxatives B. range of motion exercises of the lower extremities
D. use of suppositories C. activities of daily living (ADL)
E. no therapeutic measures are usually indicated D. heating pads to the legs
E. none of the above
42. What happens to intestinal function immediately
following spinal cord transection in the upper 48. During labor in pregnant, spinal cord-injured
thoracic area? A patient, what side effect is rather common? D
A. persitalis is lost and fecal retention occurs A. hypertension
B. function is affected B. uterine rupture
C. persitalsis is increased and severe diarrhea occurs C. sweating of the face and neck
D. retrograde peristalsis occurs D. only A and C
E. all of the above A. a paraplegia patient can often drive standard car
which has no special adaptions
49. A patient with a cauda equina injury develops B. paraplegia patients can move only specially
anemia and is started on oral iron therapy. A possible adapted vans
side effect would be B C. a paraplegia patient can drive a car only if he does
A. urinary infection not need a wheelchair
B. constipation D. a paraplegic patient can often drive a car with
C. hypoglycemia automatic transmission and hand controls
D. thrombophlebitis E. the paraplegia patient should not try to drive
E. none of the above
56. A large ischial pressure ulcer looks as if it will
50. A patient with paraplegia presents with what require surgical correction. What techniques should
appears to be a small diameter but deep ischial probably be used for adequate repairs? D
pressure ulcer. What should be considered in this A. excision of necrotic tissue and approximation of
cases? D the ulcer edge
A. osteomyelitis may be present B. excision of necrotic tissue followed by a split-
B. approximation and suturing of the ulcer edges thickness graft
C. a sinogram may provide useful information about C. rotation of a full-thickness flap
the ulcer D. rotation of full-thickness flap plus excision
D. only A and C E. none of the above
E. all of the above
Directions: For each of the incomplete statements
51. The most common upper extremity contracture in below, ONE or MORE of the numbered
the C6 quadriplegia patient is B completions is correct. In each case select.
A. adduction of the arm A. if only 1,2 and 3 are correct
B. elbow flexion B. if only 1 and 3 are correct
C. supination of the forearm C. if only 2 and 4 are correct
D. extension of the shoulder D. if only 4 is correct
E. finger extension E. if all are correct

52. Which technique for controlling lower extremity 57. Areas of concern in the definitive management of
spasticity is relatively contraindicated in the young spinal injury include E
quadriplegia male? E A. restoration of normal alignment of the spine
A. use of diazepam B. attainment of complete stability of the injured area
B. use of baclofen of the spine
C. use of dantrolene sodium C. decompression of neurologic structures that are
D. use of phenol motor point blocks being progressively compressed
E. use of intrathecal alcohol block D. early rehabilitation of the patient

53. For severe spasticity of the hip adductor muscles 58. With a suspected cervical fracture, which x-rays
the best treatment is often E should be obtained initially? B
A. diazepam 1. anteroposterior films
B. baclofen 2. flexion and extension views
C. dntrolene sodium 3. lateral views
D. surgical crushing of the obtructor nerve 4. fluoroscopy of the cervical spine in motion
E. surgical sectioning of the obtructor nerve
59. After the patient with a suspected cervical injury
54. Techniques used to manage spasticity in the arrives in the hospital emergency unit, the initial
spinal-cord-injured patient include E action should be C
A. medication 1. a complete neurological examination
B. proper bowel and bladder care 2. placement of the head a halter traction with
C. passive movement sandbags put beside the head
D. prompt treatment of infection 3. complete x-ray of the spine
E. all of the above 4. immobilization of the head and neck in a cervical
collar
55. Which of the following is true?
60. A patient is involved in an automobile accident 4. forcing oral fluids to increase intravascular
and sustains an injury to the neck. He complains of volume
some tingling in the hands. Appropriate initial
therapy at the scene of the accident would include :C 66. Goals in the management of neurogenic bladder
1. maintaining of neck in flexion include: E
2. strapping the back and neck to a rigid board before 1. prevention of infection
moving the patient 2. prevention of bladder overdistention
3. turning the head slightly to one side to prevent 3. prevention of local damage to the urethra
aspiration and bladder
4. maintaining the neck in extension 4. making the patient catheter-free as soon as
possible
61. Which of the following is/are true of the
Circolectric bed for the acute treatment of the 67. Possible difficulties from use of indwelling
spinally injured patient? D catheter with balloon include: E
1. the bed should be used because it permits safe 1. insidious deflation of the balloon
turning of the patient 2. occlusion of the catheter lumen by pressure
2. in the head-down position, the weight of the head within the balloon
provides adequate traction force on the neck, and 3. occlusion of the catheter lumen by an
artificial traction ca n be eccentrically inflated balloon
3. the bed is quite portable 4. failure of the balloon to deflate properly
4. the bed is generally felt to be contraindicated for
use in the spinal cord-injured patient 68. Advantages to use of intermittent catheterization
over an indwelling catheter include: B
62. Findings in a patient at the C2 spinal cord level 1. a lower rate of infection with intermittent
could include E catheterization
1. paralysis of the arms and legs 2. not having to insert the intermittent catheter
2. neurologenic bowel and bladder with such aseptic technique as is used for the
3. respiratory insufficiency indwelling catheter, since the intermittent
4. Horner’s syndrome catheter is in place for so short a time
3. intermittent catheterization permits the
63. Following the period of spinal shock there urethral mucosa to gradually become
develops the period of reflex automatism of the accustomed to a foreign body
isolated cord. This characterized by A 4. there are actually no major advantages of
1. hyperreflexia intermittent catheterization. An indwelling
2. rigidity catheter should generally be used
3. spasticity
4. athetosis 69. In a patient with urinary retention owing to
structural resistance of peripheral portions of the
64. Which of the follwing is/are true of phantom urinary tract, what might be appropriate therapy? B
sensations in the spinal cord-injured patient? D 1. tranurethral resection of an enlarged prostate
1. there may be a burning anf tingling sensation 2. electrical stimulation of the bladde
below the level of the lesion 3. transurethral resection of the bladder neck
2. the phantom sensations are usually most 4. cutaneous vesicostomy
troublesome in the early stages following spinal
injury 70. Major factors in the formation of upper urinary
3. the phantom sensations may remain on a relatively tract stones in the paraplegic patient include: B
permanent basis in cauda equina injurues 1. infection
4. the injury usually gives the sensation that the leg is 2. hyperuricemia
in flexion 3. hypercalciuria
4. hyperglycemia
65. A quadriplegia patient develops postural
hypotension. How might this be combated? A 71. Causes of the anemia noted with chronic renal
1. use of elastic wraps around the legs failure usually include: A
2. use of an abdominal binder 1. blood loss
3. vigorous exercise 2. hemolysis bone marrow defficiecies
3. vomiting and diarrhea
3. excessive flexor and adductor spasms may
72. Tests that are useful indetecting diminished renal be a major obstacle to the penile penetration
function include : E 4. in the complete paraplegic, the normal
1. urea clearance orgasmic sensation occurs with ejaculation
2. creatinine clearance
3. ]careful 79. Commonsites of pressure ulcers include the :E
4. blood urea level 1. sacrum
2. femoral trochanters
73. Which of the following might be beneficial in 3. lateral malleoli
obtaining evacuation of the bowel during the period 4. ischial tuberosities
of spinal shock? D
1. oral laxatives 80. Before attempting surgical correction of a
2. digital rectal stimulation pressure ulcer, which of the following should be
3. rectal suppositories required? :E
4. none of the above 1. control of active infection
2. the ulcer should be in the process of healing
74. Complications that are not infrequent in patients 3. correction of anemia
with conus-cauda injuries include: A 4. the general condition of the patient should
1. hemorrhoids be as good as possible
2. chronic constipation
3. anal fissures 81. Appropriate treatmenbt of pressure ulcers
4. rectal carcinoma includes: A
1. removal of necrotic tissue
75. What therapeutic measures can be employed to 2. thorough cleansing of the ulcer
help the quadriplegic patient achieve the goal of 3. use of a heat lamp over the ulcer
bowel evacuation 4. removing pressure from the ulcer by means
following spinal shock? A of a circular rubber ring
1. mild laxatives
2. digital rectal stimulation 82. Complications of chronic pressure ulcers can
3. glycerine or bisacodyl suppositories inclyde: E
4. enemas 1. osteomyelitis
2. amyloidosis
76. Which of the following are true of te quadriplegic 3. sepsis
female? C 4. anemia
1. menstruation ceases permanently
2. orgasm is absent in complete cord lesions 83. Activities which hould be inclyde in the physical
3. pregnancy must be aborted if it occurs therapy program of the spinal cord injured patient
4. the quadriplegic female can continue to be are: E
sexually active 1. strengthening exercises of intact muscle
groups
77. Techniques that may help the quadriplegic malew 2. range of motion exercise
to establish sexual relations include: E 3. transfer activities
1. experimenting with alternate methods of 4. ambulatory activities
sexual intercourse
2. emptying the bladder prior to intercourse 84. Which might be appropriate features on the
3. constricting the base of the penis if erctions wheelchair for a C7 quadriplegic patient? B
are short-lasting 1. detachable arm rests
4. taping a catheter back along the shaft of the 2. large drive wheel in the front
penis if an indwelling catheter must be worn 3. desk arm type of armrests
4. full reclining heavy duty chair
78. Which of the following is/are true/ A
1. reflex penile erctions can occur following 85. In which of the following might a spinal cord
spinal shock in a quadriplegic patient patient participate? E
2. in the spinal cord patient, ejaculation often 1. athletics
occurs into the bladder in a retrograde 2. recreational activities
fashion 3. educational activities
4. vocational activities
CHAPTER 8 7. Aspirin in divided doses is the most satisfactory
RHEUMATOLOGY drug for helping suppress JRA. What aspirin dosage
range is most often recommended? C
Directions: Each of the questions or incomplete A. 5-15 mg/kg daily
statements below is followed by five suggested B. 30-60 mg/kg daily
answer or completions. Select the single BEST C. 90-130 mg/kg daily
D. 250-300 mg/kg daily
1. Juvenile rheumatoid arthritis (JRA) differs from the E. 310-340 mg/kg daily
adult form of rheumatoid arthritis in that D. 8. After 10 years of JRA what percentage of patients
A. high fever, rash and uveitis are more have regained an excellent level of function? D
common in JRA A. 10-20%
B. subcutaneous nodules and rheumatoid factor B. 30-40%
are more common in JRA C. 50-60%
C. marked leukocytosis in JRA D. 70-80%
D. only A and C E. 90-100%
E. all of the above
9. What most typifies the subcutaneous nodule found
2. The onset of JRA may E in rheumatoid arthritis? B
A. be polyarticular with four or more joints A. synovial tissue
involved B. chronic inflammatory
B. be acute or fulminating with systemic C. nodular areas of glycogen deposition
manifestations but without arthritis D. absence of necrosis
C. involve one joint or only a few joints E. necrotic muscle in the nodule
D. only A and C
E. all of the above 10. Subcutaneous nodules are associated with
rheumatoid arthritis. In what percentage of patients
3. Common initial sites of involvement in JRA include with classical rheumatoid arthritis do rheumatoid
the A subcutaneous nodules develops? C
A. knees, wrists, ankles, and neck A. 3 to 7%
B. hip, elbows, shoulders and sacroiliac joints B. 10 to 15%
C. hands, feet and temporomandibular joints C. 20 to 25%
D. costovertebral joints and facet joints of spine D. 50 to 60%
E. JRA randomly involves the joints and there E. 85 to 90%
are no most common sites of initial
involvement 11.Nerve compression syndromes may occur in
rheumatoid arthritis. Which is the most common? B
4. Asymptomatic sacro-iliitis may occur as an x-ray A. ulnar nerve compression at Guyon’s canal
finding in about 24% of patients with JRA. The B. carpal tynnel syndrome
presence of sacro-iliitis correlates with E C. tarsal tunnel syndrome
A. disease onset after 10 years of age D. ulnar nerve compression at the elbow
B. hip involvement E. thoracic outlet syndrome
C. presence of rheumatoid factor
D. only A and C 12. A useful laboratory test for following the progress
E. all of the above of a case of rheumatoid arthritis is the A
A. sedimentation rate
5. A possible major complication of JRA that must be B. latex fuxation test
considered in evaluating each patients is A C. serum iron concentration
A. iridocyclitic D. leukocytosis
B. subcutaneous nodules E. none of the above are useful in following the
C. skin rash course of the disease
D. encephalitis
E. peripheral neuropathy 13. A 40-year-old female with rheumatoid arthritis is
able to perform all of her daily activities with the
6. The approximately incidence of positive rheumatoid exception of lifting her heavy bags of groceries. She is
factor detection in JRA is A also unable to continue her golfing and bowli8ng, into
A. 20% which functional class of the ARA (American
B. 40% Rheumatism Association) might this patient fall? B
C. 60% A. I
D. 80% B. II
E. 100% C. III
D. IV
E. None of the above
20. A patient with osteoarthritis has undergone a total
14. A 53-year-old male with rheumatoid arthritis hip replacement. Which physical modality should not
demonstrates the following findings: (1) osteoporosis be used over the joint implant? D
seen on x-rays: and (2) no joint deformities, but A. short wave diathermy
limitation of mobility in the hand and elbows. Into B. ultrasound
which anatomical stage of the ARA classification does C. microwave
this patient fall? B D. both A and C
A. I early E. all of the above
B. II moderate
C. III severe 21. A patient with arthritis is advised to take regular
D. IV terminal hot tub baths to help diminish stiffness and pain. The
E. None of the above approximate water temperature that should be used is
C
15. The nonmedicinal management of the patient with A. 60F
rheumatoid arthritis which of the following? E B. 80F
A. Systemic rest C. 100F
B. Articular rest D. 120F
C. Physical rest E. 140F
D. Proper nutrition
E. None of the above 22. One of the most useful modalities for treating the
rheumatoid hand involves A
16. In the treatment of rheumatoid arthritis with A. paraffin
salicylates, salicylism is used as a guideline in helping B. hot warm bottles
to individualize the dose. In an adult, the usual first C. ultrasound given under water
manifestation of salicylism is E D. short wave diathermy
A. peptic ulcer E. heating pads
B. anemia
C. centrally induced hyperventilation 23. Which of the below exercises is not indicated for
D. psychosis used in the average arthritic patient? C
E. tinnitus A. passive range motion
B. active range of motion
17. Physical therapy modalities can be useful for the C. rapidly performed coordination
arthritic patient at home. The patient should be D. isometric strengthening
instructed in how to properly use these treatment E. active-assistive
modalities. What is the major and most frequent
hazard encountered in the home treatment program? 24. Which physical heating modality might be of most
D benefit to the arthritic patient with painful degenerative
A. fractures from too vigorous exercise arthritis to the hip? C
B. skin infection from improper use of massage A. short wave diathermy
C. neck dislocation of improper use of traction B. microwave
D. burns from improper use of heat C. ultrasound
E. deformities aggravated by homemade splints D. hot packs
E. ultraviolet
18. What might be a useful treatment for the arthritic
patients to help alleviate morning stiffness? D 25. The common deformities seen in the arthritic knee
A. sleeping on a heating pad include: E
B. taking a hot bath before retiring for bed A. knee flexion contractures
C. sleeping with hot water bottles B. subluxation
D. using an electric blanket C. external rotation
E. none of the above D. both A and C
E. all of the above
19. A patient with osteoarthritis develops a knee
effusion. Which physical therapy modality should not 26. The most common age period for onset of
be used over the knee? D ankylosing spondylitis is C
A. hot packs A. birth to 5 years
B. ice massage B. 5 to 20 years
C. infrared C. 20 to 40 years
D. microwave D. 40 to 60 years
E. ultrasound E. over 60 years

27. The joints which usually first show x-ray


abnormalities in ankylosing spondylitis are the D
A. vertebral facet joints 34. Approximately what percentage of patients with
B. metacarpophalangeal joints psoriasis develop arthritis? A
C. interphalangeal joints A. 5%
D. sacroiliac joints B. 25%
E. hip joints C. 50%
D. 75%
28. What percentage of cases of ankylosing E. 95%
spondylitis have involvement of the peripheral joints at
some stage in the disease? C 35. The joints most commonly involved in psoriatic
A. 0% arthritis are the D
B. 15% A. hip joints
C. 35% B. knee joints
D. 75% C. sacroiliac joints
E. 100% D. distal interphalangeal joints of the fingers
and toes
29. At times the onset of ankylosing spondylitis is in E. carpormetacarpal joints
the peripheral joints. In what percentage of cases
does this occur? C 36. Which of the following medications is
A. 5% contraindicated in the treatment of psoriatic arthritis?
B. 10% D
C. 20% A. systemic coritcosteriods
D. 50% B. gold
E. less than 1% C. intraarticular corticosteriods
D. antimalarials
30. A patient with ankylosing spondylitis complains of E. methotrexate
pain radiating into the lower extremities. A likely
explanation for this symptoms is B 37. What is the most serious complication of
A. the patient has probably herniated a nucleus rheumatic fever? A
pulposus and has nerve root pain A. carditis
B. the pain is probably originating from the B. arthritis
sacroliac joint. C. chorea
C. A bone spur is probably causing pressure on D. pharyngitis
a nerve root in the back E. pneumonitis
D. Both A and C
E. All of the above 38. How commonly do arthritis and arthralgia occur at
some time in the course of systemic lupus
31. In what percentage of cases of ankylosing erythematosus (SLE)? E
spondylitis is the course of the disease relatively A. in less than 5% of cases
benign over 1—20 year period? C B. in 20% of cases
A. 5% C. in 50% of cases
B. 20% D. in 75% of cases
C. 40% E. in 90% of cases
D. 80%
E. 95% 39. The most frequent cause of death in the patient
with SLE is E
32. In what percentage of patients with ankylosing A. pericarditis
spondylitis is there rapid progression to severe B. stroke
disability? B C. myocardial infraction
A. less than 1% D. malignancy
B. 5% E. nephritis
C. 25%
D. 50% 40. patients with SLE occasionally develop
E. 70% psychoses. In what percentage of cases of SLE would
psychosis at some time be present? D
33. Usually the most appropriate analgesic for use in A. less than 1%
patients with ankylosing spondylitis is A B. 5%
A. salicylates C. 20%
B. indomethacin D. 50%
C. acetaminophen E. 90%
D. codeine
E. meperidine 41. In patients with rheumatoid arthritis, lupus
erythematous (LE) cells may occasionally be
demonstrated. Which of the following statements is D. 85%
true about this group of patients? E E. 100%
A. most go on to develop the typical lupus
nephrtiis 48. Which of the following are features of the
B. many develop the typical lupus vasculitis peripheral neuritis found in polyarteritis? A
C. the lupus dermatitis is common in these 1. paresthesias
patients 2. burning pains
D. all of the above 3. absence of motor weakness
E. these patients do not generally go on to 4. primary involvement of the upper
develop features of SLE extremities
5. asymmetry
42. The prognosis of drug-induced SLE B
A. is the same as for SLE A. only 1,2, and 5
B. is better than that for SLE B. only 3.4 and 5
C. is worse than that for SLE C. only 1,2,3 and 4
D. is quite variable, with the prognosis D. only 4
depending on the severity of the renal E. all are correct
involvement
E. is quite variable depending on the amount of 49. The major cause of death in polyartheritis is D
cardiac disease present A. cardiac disease
B. stroke
43. A patient being treated with procainamide C. pneumonia
develops arthritis and hepatosplenomegaly after six D. renal disease
months of using the drug. A likely explanation is D. E. gastrointestinal bleeding
A. the patient has developed rheumatoid
arthritis 50. The most frequent initial symptom in polymyositis
B. the patient is suffering from a viral arthralgia and dermatomyositis is B
secondary to hepatitis A. skin rash
C. the patient suffers from a drug-induces B. weakness of the legs
psoriatic arthritis C. joint and muscular pain
D. the patient has develop drug-induced SLE D. weakness of the arms
E. none of the above E. dysphagia

44. the 10years survival rate after a diagnosis of SLE 51. The most consistently weak muscle in
has been made is approximately D polymyositis are the B
A. less than 10% A. neck flexor
B. 15% B. proximal muscles of the lower extremities
C. 25% C. facial muscles
D. 50% D. hand intrinsic muscles
E. 80% E. extrocular muscles

45. the prevalence of renal disease in polyartheritis is 52. In what percentage of cases of polymyositis and
D dermatomyositis would one expect to see the
A. 5% presence of Raynaud’s phenomenon? C
B. 25% A. 0%-if it occurs, it indicates that the diagnosis
C. 50% was in error
D. 75% B. 5%
E. 100% C. 30%
D. 75%
46. The most frequent area of heart involvement in E. 100% - if it is not present, the diagnosis must
polyartheritis is in the B be reevaluated
A. endocardium
B. myocardium 53. A nearly pathognomonic rash seen in
C. pericardium dermatomyositic is D
D. there is equal involvement of all of the above A. a butterfly rash on the face
E. polyartheritis does not involve the heart B. anerythematous rash on the elbows
C. a scaly rash on the anterior chest wall
47. What percentage of cases of polyartheritis will D. a dusky lilac suffusion on the upper eyelids
develop a peripheral neuritis? C E. a periungual erythematous and scaly rash
A. 5%
B. 25%
C. 50%
54. In a 40 years old male with dermatomyositis what B. renal
is the probability of eventually discrovering a C. rectal
malignant lesion? C D. neurological
A. 100% E. esophageal
B. 75%
C. 50% 62. Osteoarthritis is D
D. 25% A. an inflammatory disease of bone
E. 5% B. a degenerative disease of bone
C. an inflammatory disease of articular cartilage
55. The initial treatment of choice for polymyositis is E D. a degenerative disease of articular cartilage
A. methotrexate E. all of the above
B. salicylates
C. gold 63. On of the least involved portions of the joint in
D. phenylbutazone osteiarthritis is the A
E. croticosteriods A. synovial tissue
B. bone
56. The most common form of onset of sclerodema A C. cartilage
A. Raynaud’s phenomenon or swelling and D. ligaments
thickening of hands and fingers E. tendons
B. Arthralgias
C. Severe muscle weakness 64. The term Heberden’s node refers to the
D. Visceral involvement (dysphagia and osteoarthritic changes occurring at which joint? E
intestinal involvement) A. metacarpophageal
E. Renal disease B. tarsal-metatarsal
C. proximal interphalangeal
57. What is the proper sequence of skin changes in D. carpometacarpal of the thumb
sclerodema (earliest to latest)?C E. distal interphalangeal
A. indurative, atrophic, and edematous
B. atrophic, indurative and edematous 65. Bony enlargement of the proximal interphalangeal
C. edematous, indurative and atrophic joint of finger is prenent in a patient with advanced
D. indurative, edematous, and atrophic osteoarthritis. The name of this enlargement would be
E. edematous, atrophic, and indurative aC
A. Heberden’s node
58. The incidence of Raynaud’s phenomenon on B. Schmori’s node
patient with sclerodema is approximately A C. Bouchard’s node
A. 90% D. Fibro-osseus node of Schmidt
B. 70% E. There is no specific name for the nodular
C. 50% enlargement
D. 30%
E. 10% 66. Degenerative changes in joints become more
frequent with advancing age. Generally at what age
59. The most frequent x-ray feature seen in may degenerative changed first be seen in joints
sclerodema is C autopsy?B
A. narrowing of the intervertebral disc A. first decade of life
B. erosion of the carpal bones B. second decade of life
C. absoption of the tufts of the distal phalanges C. third decade of life
D. spotty osteoporosis underlying areas of skin D. fourth decade of life
changes E. fifth decade of life
E. loss of articular cartilage of involved joints
67. The chief symptom of osteoarthritis is usually D
60. A patient with sclerodema develops proximal A. edema
muscle wekanes. A likely explanation is that the B. fever
patient has developed A C. muscular weakness
A. sclerodematomyositis D. pain
B. limb-girgle muscular dystrophy E. joint deformity
C. duchenne muscular dystrophy
D. viral myositis 68. The pain osteoarthritis is usually B
E. myotonic dystrophy A. a burning sensation
B. aching in character
61. The most common form of visceral involvement in C. a sharo, well localized pain
sclerodema is E D. a sharp, radiating pain
A. cardiac E. a combination of all of the above
B. measuring serum uric acid levels
69. The most consistency helpful laboratory C. measuring urate excretion in the feces
abnormality in confirming the diagnosis of D. examining the synovial fluid
osteoarthritis is E E. assessing dietary habits
A. an elevated sedimentation arte
B. leukocytosis 77. The synovial fluid a patient with possible acute
C. a normocytic, normochronic anemia gouty arthritis contains no monosodium urate crystals.
D. rheumatoid factor Your next move is to D
E. none of the above A. evaluate the patient for other causes of acute
arthritis
70. X-ray changed seen in osteoarthritis include E B. begin the patient on salicylate therapy
A. joint space narrowing C. reassure the patient and have him return for
B. marginal osteophyte formation a recheck in one week
C. subchondrial bony sclerosis D. aspirate more synovial fluid for examination
D. both A and C E. get anteroposterior and lateral x-rays of the
E. all of the above involved joints.

71. The acute attack of gouty arthritis is triggered by E 78. Which would probably be the preferred oral
A. a food allergy medication for the treatment of acute gouty arthritis?
B. periarticular vasoscpasm D
C. hyperuricemia A. aspirin
D. a decrease in the blood glucose level B. cochicine
E. sodium urate crystals C. allopurinol
D. phenylbutazone
72. A patient with chronic gouty arthritis develops E. methotrexate
carpal tunnel syndrome. A likely explanation is that B
A. a urate vascular plaque is compressing the 79. The drug allopurinol is used in the treatment of
median nerve hyperuricemia. What is its mechanisms of action? D
B. a tophus is compressing the median nerve A. it increases uric acid excretion in the urine
C. the patient has diabetic neuritis secondary to B. it increases uric acid excretion in the feces
the gout C. it increase the solubility of uric acid in the
D. the patient has satumine gout serum
E. none of the above D. it inhibits xanthine oxidase
E. it inhibits white cells migration to urate
73. The major component of a tophus in gout is A crystals
A. sodium urate crystals
B. lipid 80. A 40 year old male develops dermatomyositis. He
C. collagen should be also closely evaluated for the presence of D
D. calcium monohydrate A. diabetes
E. bile salts B. hyperparathyroidism
C. Addison’s disease
74. A patient being treated for hypertension develops D. Malignant tumor
acute gout. A likely cause is that C E. Hypertriglyceridemia
A. the antihypertensive medication decreased
the uric solubility 81. A patient presents with complaints of stiffness and
B. the patient also had leukemia discomfort in the low back. The patient’s urine has
C. the diuretics used to treat the hypertension been noted to turn black upon standing. Also x-rays of
D. in an attempt to food a low-sodium diet, the the spine show calcification of the intervertebral discs.
patient ate foods with excessive amount of The likely disganosis is E
purines A. ankylosing syndrome
E. none of the above B. Reiter’s syndrome
C. Wilson’s disease
75. the male female ration in primary gout is D. Pseudogout
approximately A E. Ochronosis
A. 9:1
B. 3:1 82. The organism most often found to cause
C. 1:1 infectious pyogenic arthritis in children is A
D. 1:3 A. Staphylococcus aureus
E. 1:9 B. Diplococcus pneumonia
C. Streptococcus pyogenes
76. The best way to diagnose gout is by D D. Neisseria gonorrhea
A. history E. Hemophilus influenzae
B. if only 1 and 3 are correct
83. The organism most often found to cuse infectious C. if only 2 and 4 are correct
pyogenic arthritis in the adult is D D. if only 4 is correct
A. Staphylococcus aureus E. if all are correct
B. Diplococcus pneumonia
C. Streptococcus pyogenes 89. The treatment program in juvenile rheumatoid
D. Neisseria gonorrhea arthritis includes E
E. Hemophilus influenzae 1. joint rest during acute attacks
2. splints to correct deformities
84. A male develops fever, polyarticular arthritis 3. heart
urethritis and conjunctivitis. A lkely diagnosis would be 4. exercises
C
A. gonococcal arthritis 90. characteristics of the anemia found in rheumatoid
B. rheumatoid arthritis arthritis include C
C. Reiter’s syndrome 1. a good response to the
D. Ankylosing spondylitis administration of vitamin B-12
E. Psoriatic arthritis 2. failure to response to the
administration of iron
85. A knee x-ray is taken several months following 3. association with macrocytic red
acute trauma to the knee. A calcific debsity is noted in blood cells
the area of the medical collateral ligament. A likely 4. association with normochronic red
diagnosis is D blood cells
A. juvenile rheumatoid arthritis
B. ankylosing spondylitis 91. Which statements about the sedimentation rate
C. chrondrocalcinosis are true? A
D. tuberculosis of the spine 1. the sedimentation rate normally
E. the renal disease found in gout rises slightly in the process of aging
2. the sedimentation rate is usually
86. A patient presents with chest pain, tenderness and normal in primary osteoarthrtis
firm swelling in the area of the left third costochrondral 3. the disorder polymyagia rhematica
junction. No history of trauma can be obtained. Deep is distinguished by a moderately to
breathing increases the pain . a likely diagnosis is A markedly elevated sedimentation
A. dermatomyositis rate
B. sclerodema 4. the sedimentation rate is elevated in
C. Chrondrocalcinosis fibrositis
D. Pellegrini-Steieda syndrome
E. Jaccoud’s arthritis 92. The LE phenomenon and antinuclear factors are
characteristics of SLE. Which of the following are
true? C
87. A patient presents with chest pain, tenderness and 1. antinuclear factors and the LE
firm swelling in the area of the left third costochrondral phenomenon are not found in
junction. No history of trauma can be obtained. Deep patients with rheumatoid arthritis
breathing increases the pain. A likely diagnosis is A 2. as a group, rheumatoid arthritis
A. Tietze’s syndrome patient with a positive LE cell test
B. Atypical angina tend to have more pronounced
C. Pneumonitis systemic involvement
D. Rib fracture 3. the LE phenomenon in a patient will
E. Scheuemann’s disease go on to develop features of SLE
4. antinuclear factors have been
88. In acute arthritis of the wrist, what form of reported in various disorders
therapeutic exercise should be utilized? A characterized by chronic
A. passive range of motion inflammation
B. isometric strengthening
C. isotonic strengthening 93. Which statements about the serum complement
D. isokinetic strengthening level are true? E
E. none of the above is indicated in acute 1. it is usually normal or somewhat
arthritis elevated in rheumatoid arthritis
2. it is decreased in patients with
Directions: for each one of the incomplete statements vasculitits
or questions below, ONE or MORE of the numbered 3. it is often elevated early in the
completions is correct. In each case select. course of rheumatic fever
A. if only 1,2 and 3 are correct
4. it is reduced in most patients with 3. osteoporosis
SLE 4. carbohydrate intolerance

94. Which of the following are useful clinical features 99. Intraarticular corticosteroid injection is
in helping to make the diagnosis of rheumatoid occasionally use in treating the patient with
arthritis? A rheumatoid arthritis. Which of the following are
1. slowly progressive, symmetrical indications for considering the use of intraarticular
polyarthritis corticosteriod injections? C
2. prolonged early morning stiffness 1. infection in or near the joint
3. periarticular swelling of soft tissues 2. inflammation of one or only a few
4. weakness of the trunk musculature peripheral joint
3. severe juxtaarticular osteoporosis
95. A patient with rheumatoid arthritis develops acute 4. contraindication of systemic gold,
arthritis of the wrist. Which of the following measures cortisone, or other therapy
might be indicated in the treatment? A
1. daily passive range of motion 100. The objectives for the use of physical therapy in
exercise arthritis include E
2. superficial heat 1. helping to relive symptoms
3. salicylate analgesic 2. shortening the period of disability
4. articular rest by placing the forearm from the disease
and hand in a plaster cast for two 3. preventing or helping to correct
weeks deformities
4. improving function
96. Which of the following statements is true about the
use of gold therapy in rheumatoid arthritis? C 101. A patient with rheumatoid arthritis is referred to
1. gold therapy can occasionally the physical therapy department with a prescription,
reverse some of the damage done which reads “physical therapy daily’. What is your
by the arthritis response to his prescription? D
2. gold therapy should not be used in 1. the prescription is satisfactory in
the patient with SLE that it gives the therapist freedom to
3. the most serious toxic side effect of develop the most adequate therapy
gold therapy is dermatitis program
4. the best results from gold therapy 2. the prescription is inadequate in
occur when the drug is used that the patient should be treated on
relatively early in the course of the a twice daily basis
illness 3. the prescription is adequate as far
as it goes but occupational therapy
97. Systemic corcosteriods are occasionally useful in should also be specified.
controlling rheumatoid arthritis. Which of the following 4. the prescription lacks sufficient
statements are true concerning the use of detail to be acceptable
corticosteriods? B
1. patient who are candidates for 102. Some indication for the use of therapeutic
steroids therapy are those with exercise in arthritis include E
active disease who are not 1. preservation or improvement of joint
responding to the usual measures range of motion
of treatments 2. preservation or improvement of
2. corticosteroids therapy can muscular strength and endurance
occasionally reverse some of the 3. maintenance of useful ambulation
damage caused by the arthritis for as long as possible
process 4. maintenance of upper extremity
3. severe rheumatoid vasculitis and function
hypersplenism are relative
indication for the use of systemic 103. In a patient with ankylosing spondylitis, which
corticosteriods exercise are most indicated? C
4. the dosage of corticosteriod used in 1. knee-chest exercises
rheumatoid arthritis should 2. back hyperextension exercises
generally be rather large. 3. hip range of motion exercises
4. deep breathing exercises
98. Which of the following are potential side effects to
the use of systemic corticosteriods? E 104. Some areas in which the occupational therapist
1. gastrointestinal ulcerations can help the arthritic patient include E
2. fatty deposition
1. providing appropriate splints for the disease to flare up to 30% of the
upper extremities other treated patients.
2. helping the patient maintain or
regain upper extremity strength and 110. Which of the following statements about psoriatic
range of motion arthritis are true? D
3. helping to evaluate the patient’s 1. tests for rheumatoid factor are
vocational potential usually positive
4. helping the patient to achieve 2. subcutaneous nodules are found in
independence in activities of daily the majority of cases
living 3. x-ray features are identical to those
seen with rheumatoid arthritis
105. Which of the following statements about 4. the spine and sacroiliac joints can
ankylosing spondylitis are true? D be involved
1. ankylosing spondylitis is most
common in females 111. Laboratory test that are commonly found to be
2. the eitiology of ankylosing abnormal in psoriatic arthritis include C
spondylitis is thought to be 1. rheumatoid factor
infectious 2. uric acid
3. the onset of symptoms is often 3. IgG levels
abrupt 4. sedimentation rate
4. the earliest symptom is usually low
back pain 112. Which of the following are true about the arthritis
seen in rheumatoid fever? B
106. With cervical spine involvement in ankylosing 1. joint involvement becomes more
spondylitis, which of the following are potential common in the older group of
problems? A patients with rheumatic fever
1. cervical radiculopathy 2. carditis and chorea parallel the
2. cervical spine fractures presence of arthritis
3. spinal cord injury 3. the arthritis usually occur early in
4. cauda equina injury the course of the illness
4. the arthritis often results in mild joint
107. Which of the following are typical x-ray changes deformities
seen in akylosing spondylitis
1. slight to moderate narrowing of the 113. Which of the following can be seen in Sjogren’s
disc spaces syndrome? A
2. squarinf of the vertebral bodies 1. rheumatoid arthritis
3. ossification of the connective tissue 2. myopathy
around the vertebrae 3. the sicca complex
4. ossification of the nucleus pulposus 4. predominance of the disease in
males
108. Sacroiliac x-ray changes are characteristic of
ankylosing spondylitis but can laso seen in other 114. The fibrositis syndrome B
disorder. Which of the following disorders can 1. is characteristic by areas of
occasionally produce sacroiliac x-ray changes? A localized point tenderness over
1. psoriasis muscles
2. ulcerative colitis 2. is associated with increased in the
3. reiter’s disease sedimentation rate
4. gout 3. often presents complaints of pain
and stiffness
109. X-ray therapy can provide symptomatic relief in 4. is a wastebasket term that has little
about 60% of patients with ankylosing spondylitis yet clinical value.
is rarely used as a dorm of treatment for this disorder.
Why is this? B 115. Which of the following are typical of the fibrositis
1. bone marrow aplasia follows the x- syndrome? A
ray therapy 1. increase in symptoms in damp and
2. radiation burns are common in cool weather
patients treated with the x-ray 2. complaints of chronic fatigue
therapy 3. relief of symptoms with the use of
3. there is a higher risk of developing heat
leukemia following the irradiation 4. constant level of discomfort
4. while the x-ray therapy may help up persisting for periods of several
to 60% of patients, it may cause the weeks
1. hydralazine
116. Features consistent with the diagnosis of 2. diphenylhydantoin
polymyalgia rheumatica include C 3. procainamide
1. patients usually in the 20 to 40 4. methyldopa
years age group
2. markedly elevated sedimentation 124. Which o the following statements are true about
rate polyarteritis? D
3. pain in the proximal joints 1. females comprise the majority of
4. relief of symptoms with the use of cases
systemic corticosteriods 2. the disease mainly involves large
arteries
117. Which of the following are consistent with the 3. the disease occurs primarily in the
diagnosis of polymyalgia rheumatica? B elderly
1. pain on motion 4. the disease most frequently
2. muscle weakness involves the kidneys and heart
3. abrupt onset
4. usually a normal hemoglobin level 125. The initial symptoms of polyarteritis may include
E
118. Which of the following are features of SLE? A 1. fever of unknown etiology
1. most commonly found in females 2. renal disease
2. usually begins in the third to fourth 3. cardiac disease
decades 4. peripheral neuritis
3. may present with a highly variable
clinical picture 126. Which of the following statements are true about
4. follows a stereotyped pattern after polymyositis and dermatomyositis? E
its onset 1. women are involved twice as often
as men
119. Very common initial manifestation of SLE include 2. may be associated with malignant
E disease
1. fever 3. may occur at almost any age
2. fatigue and malaise 4. may occur in conjunction with one
3. acute polyarthritis of the other connective tissue
4. erythematous skin eruption diseases

120. Which of the following are typical of the arthritis 127. Patients with polymyositis occasionally complain
seen in SLE? D of pain. What are the most common sites of pain? B
1. absence of morning stiffness 1. shoulders
2. ankylosis and contracture 2. calves
3. absence of subcutaneous nodules 3. upper back
4. frequent involvement of the 4. lower back
proximal interphalangeal and
metacarpophalangeal joints of the 128. Which of the following are true about
hands polymyositis dermatomyositis? D
1. contractures are relatively common
121. Neurologic involvement which can occur in SLE early in the course of the disease
includes E 2. severe atrophy of muscles is
1. peripheral neuritis common
2. seizures 3. the typical skin rash is present in
3. cerebral infraction 5% of cases
4. transverse myelitis 4. calsinosis of the muscles occurs
most often in children
122. Laboratory abnormalities in the patient with
systemic lupus erythematous can include E 129. Features which can occur when dermatomyositis
1. normochronic normocytic anemia is associated with malignant tumor included E
2. presence of LE cells 1. generally poor therapeutic response
3. reduced levels of serum to corticosteriods
complement 2. age of over 40 years for most
4. presence of antinuclear antibodies patients
3. the muscle disease takes the form
123. Certain drugs have been implicated as inducing of a florid dermatomyositis
a clinical picture typical of SLE. Which of the following 4. the neoplasm is usually a
drugs can produce a drug-associated SLE? E carcinoma
2. they may develop rather rapidly with
130. Which of the following statements is true about associated redness, swelling
scaleroderma? E tenderness and aching
1. females are affected two to three 3. they are more common in women
times as often as males 4. they are seen in the proximal
2. it is more common in coal miners interphalangeal joints
3. it is relatively uncommon in
childhood 137. joints commonly involved in degenerative arthritis
4. there are no significant racial (osteoarthritis include the) C
differences in incidence 1. shoulders
2. cervical spine
131. Esophagela abnormalities seen in 3. wrists
cinefluoroscopic examination of the patient with 4. hips
scleroderma include A
1. decrease or absence of peristaltic 138. Joints frequently involved in degenerative
action arthritis (osteoarthritis) include the E
2. gastroesophageal reflux 1. elbows
3. dilatation of the lower portion of the 2. wrists
esophagus 3. shoulders
4. constriction of the upper portion of 4. first metacarpophalangeal joints
the esophagus
139. Which of the following are true about
132. What are the major forms of new bone formation osteoarthritis? A
seen in osteoarthritis? C 1. symptoms are uncommon before
1. ectopic muscular calcification the age of 40 years
2. subchondral eburnation 2. stiffness is usually of short duration
3. intraarticular ossification 3. bony ankylosing is uncommon
4. marginal osteophytes 4. joint effusions are relatively
common
133. Subchondral bone ‘cysts” may develop in
osteoarthritis and on x-ray may appear similar to the 140. Which of the following are true about
lesions seen in C osteoarthritis? E
1. diabetes 1. diet
2. gout 2. medication
3. pernicious anemia 3. physical therapy
4. hemophilia 4. joint rest

134. Which statements about osteoarthritis are true? 141. A patient with painful osteoarthritis of the right hip
A is referred for physical therapy. What would be
1. under the age of 45 years, males appropriate treatments? A
are involved more frequently than
females 1. use of ultrasound encouraging the
2. osteoarthritis is related to the patient to lose weight if he is overweight
presence of obesity 2. encouraging the patient to lose weight
3. after the age of 55, the hands in if he is overweight
females are more involved than 3. range of motion exercises to the
those in males involved hip
4. whites have a higher incidence of 4.using a cane in the right hand to unload
osteoarthritis than black do. stress from the hip

135. systemic findings common in osteoarthritis 142. Which of the following is true about the initial attack of
include D gout? A
1. fever 1. onset is usually abrupt
2. anemia 2. often involves the great toe
3. nausea and vomiting 3. may last a few days to a few weeks
4. none of the above 4. recovery is seldom complete

136. Which of the following statements are true about 143. Common sites for the development of gouty tophi
Heberden’s nodes? A include the A
1. they may develop gradually with 1. ear
little or no pain 2. tendons
3. patellar and olecranon bursae
4. aorta

144. Which of the following is true about the renal disease


associated with gout? E
1. other than arthritis, it is the most
common complication of gout
2. it is usually slowly progressive
3. the incidence of proteinuria varies
from 20 to 40%
4. renal failure is the cause of death in
about one fourth of patients with
gout

145. Secondary forms of gout can occur with E


1. severe alcoholism
2. starvation
3. renal disease
4. leukemia

146. Which of the following drugs can alter the serum


urate levels? A Chapter 9
1. salicylates
2. thiazide diuretics Neurology
3. phenylbutazone
Directions: Each of the questions or incomplete
4. colchicines
statements below is followed by five suggested
answers or completions. Select the single BEST
147. The Charcot neuropathic joint may occur with E answer in each case.

1. diabetes mellitus 1. A hyperactive jaw reflex maybe an indication of


2. syringomyelia bilateral pyramidal tract dysfunction. At which level
3. myelomeningocele of the central nervous system is the jaw jerk
4. tabes dorsalis
mediated? C
148. Characteristic of the Charcot joint include B A. cerebral hemispheres
1. hypermobility of the joint B. cerebellum
2. usually severe pain C. pons
3. possible combinations of D. medulla
destructive and hypertrophic E. upper cervical segments of the spinal cord
changes on x-rays
4. the patients neurological 2. A 47 year old man is being seen for a routine
examination will be normal. physical examination. His neurological examination
appears normal except that no deep tendon reflexes
can be obtained in the upper or lower extremities.
The most likely reason for this is: E
A. generalized peripheral neuropathy
B. myopathy
C. spinal cord lesion
D. this maybe a normal variant
E. this maybe normal variant

3. Recovery in a middle cerebral artery stroke is


usually most complete in the muscles around the: C
A. hand
B. shoulder
C. hip
D. ankle
E. all of the above recover equally
4. The circle of Willis provides connection between B. deep leg veins
the carotid and vertebral-basilar artery system. C. heart
Anomalies in the circle of Willis may reduce it’s D. lungs
ability to provide needed additional circulation. In the E. lymphatic system
general population, what is the approximate
prevalence of anomalies of the circle of Willis? D 11. Following stroke involving the carotid arterial
A. 1% system, the physical finding which would indicate the
B. 10% poorest prognosis is :D
C. 25% A. hemiparesis
D. 50% B. hemiplegia
E. 75% C. hemianopsia
D. altered consciousness
5. The most common cause of stroke is :D E. expressive aphasia
A. cerebral embolism
B. intracerebral hemorrhage 12. Systemic hypertension is found in approximately
C. subarachnoid hemorrhage what percentage of cases of cerebral hemorrhage?
D. cerebral thrombosis A
E. subdural hematoma A. greater than 90%
B. 75%
6. The clinical findings in the anterior horn cell C. 50%
disease would include all of the following except: C D. 25%
A. fasciculation E. less than 10%
B. loss of reflexes
C. sensory abnormalities 13. The most common symptom in intracranial
D. muscle weakness hemorrhage is :B
E. atrophy A. dizziness
B. headache
7. Following conclusion of the anterior cerebral C. confusion
artery the greatest amount of motor deficit is found in D. vomiting
the :B E. nausea
A. contralateral arm
B. contralateral leg 14. The most common cause of a saccular or “berry”
C. ipsilateral arm aneurysm is :D
D. ipsilateral leg
E. involvement will be equally severe in both A. Hypertension
legs B. connective tissue disease with vasculitis
C. Buerger’s disease
8. Complete occlusion of the basilar artery usually D. Congenital defect of the vessel wall
causes : A E. All of the above are equally commomn
A. death
B. quadriplegia 15. The usual time chosen for performing surgery
C. hemiplegia following suubarachnoid hemorrage is :D
D. a Brown-Sequard syndrome A. As soon as possible
E. no symptoms B. When the neurological lesion progresses to
show corticospinal tract involvement
9. Cerebral embolism accounts for what percentage C. C. when the neurogical lesion progresses to
of all pathologically documented strkes? C show cranial nerve involvement
A. less than 1% D. When patient is clinically stable
B. 10% E. Surgery is never indicated
C. 25%
D. 50% 16. The incidence of mortality following
E. 75% subarachnoid hemorrhage is generally about :C
A. 100%
10. The primary site of origin for cerebral embolism B. 75%
is the :C C. 50%
A. superficial leg veins D. 25%
E. 0% 23. Chronic subdural hematoma can be characterized
by :E
17 Rupture of a “berry” aneurysm primarily leads to A. recurrent headache
:B B. drowsiness
A. cerebral embolism C. convulsion
B. subarachnoiod hemorrhage D. confusion
C. cerebral thrombosis E. all of the above
D. intracerbral hemorrhage
E. there are usually no clinical findings 24. following a strke, when should the patient begin
rehabilitation :D
18. Findings in adhesive spinal arachnoiditis include A. ater three weeks of bedrest
:D B. after two weeks of bedrest
A. intense, persistent unilateral radicular pain C. after one week of bedrest
which may later become bilateral D. when the patient is medically stable
B. signs of complete spinal cord lesion E. one week after the patient is medically
C. males are more commonly involved than stabled
females and the onset is usually in middle
age 25. Areas of concern for the occupational therapist in
D. only A and C dealing with the stroke patient include :E
E. all of the above A. evaluating and instructing in the activities of
daily living
19. Causes of adhesive spinal arachnoiditis include B. assisting the patient performance of one
:E handed activities
A. trauma to the spine C. assisting in evaluating the need for various
B. spinal anesthesia self-help devices
C. meningitis D. assisting in the perceptual evaluation of the
D. only bA and C patient
E. all of the above E. all of the above

20. Following a head injury, the “posttraumatic 26. Which of the following features should be
syndrome” may develop. Common symptoms include specified in a wheelchair obtained for a stroke patient
:E :E
A. vertigo A. large drive wheel in the front to allow the
B. fatigue and sleeplessness patient to better grasp it
C. headache B. a one-hand drive chair with two concentric
D. both A and C handrims
E. all of the babove C. a full reclining model so that the patient can
lie down and rest
21. Features of acute subarachnoiod hemorrhage D. all of the above features should usually be
include :D specified
A. headache E. none of the above should usually be
B. normal level of awareness specified
C. stiff neck
D. both A and C 27. A patient is brought in for examination by his
E. all of the above family. He is 62 years old and sustained a stroke 4
years ago. He has been unable to walk since the
22. Among features of acute subdural hematoma are stroke. He underwent two months of rehabilitation at
that it :E the time of stroke and has also received other courses
A. Ussually follows severe head injury of physical therapy as an outpatient. The family feels
B. B. Is usually associated with major brain that he is now moving his leg much better and is
injury recovering from the stroke. He recently underwent
C. Is marked loss of consciousness two months of additional physical therapy elsewhere
D. Both A and C but still could not walk in the parallel bars. The
E. All of the above therapy was discontinued and the family complains
that the therapy was stopped just as the patient was
starting to show more improvement. They say that he
moves around much better at home than in the B. ventral roots of the spinal nerves
therapy department. They insist on additional C. anterior horn cells
physical therapy. Your examination reveals poor D. peripheral nerves
standing balance. Would you order additional E. corticospinal tracts
therapy? :E
A. Yes, the patient was probably improving and 32. A patient presents with a unilateral vesicular
should be given additional therapy eruption of the chestwall, which extends around the
B. Yes, although the patient was probably not chest in a narrow band. A sharp pain is also present in
improving, more therapy would be the area corresponding to the rash. A likely diagnosis
psychologically beneficial to the patient would be :B
C. Yes, more therapy would be psychologically A. eczema
beneficial to the family B. herpes zoster
D. No, the patient had recent therapy. A period C. psoriasis
of 4 to 6 months should pass before D. Guillain-Barre syndrome
additional therapy is started E. Protruded thoracic intervertebral disc
E. No, the patient and family should be
counseled with understanding and 33. After a hotn tub bath, a patient with multiple
compassion into beeter adjusting to the sclerosis complains that he is m arkedly fatigued and
stroke and its disability weakened. How would this be explained? :A
A. this is a normal finding in multiple sclerosis
28. A patient has suffered severe stroke, Over what patients
period of time might you expect that improvement in B. heat causes the multiple sclerosis to greatly
muscular control might occur? :C progress
A. in first week only C. the patient should undergo a complete
B. over the first neurological reevaluation as soon as possible
C. over the first year D. the patient should be referred to an internist
D. over three years for complete evaluation
E. some cases may recover upto five years E. this not occur with multiple sclerosis, but
post-stroke even if it did not would not warrant any
further studies.
29. Which of the following is true about the use
of hand splints in hemiplegia? :E 34. The age range in which the onset of multiple
A. they may facilitate return of function in the sclerosis most commonly occurs is :C
hemiplegic hand A. birth to age 3 years
B. dynamic splints are often useful to help B. 5 to 15 years
substitute for paralyzed muscles C. 20 to 30 years
C. all hemiplegic hands should be routinely D. 45 to 60 years
splinted to prevent contractures E. over age 60 years
D. all of the above are correct
E. none of the above is correct 35. In the United States, the approximate number of
patients with multiple sclerosis is :D
30. An elderly female stroke patient has a foot-drop A. 600
but adequate knee stability. The patient is somewhat B. 7,500
concerned about cosmesis. What might be the best C. 50,000
treament? :E D. 150,000
A. surgical fusion of ankle E. 2,000,000
B. long leg brace
C. double upright short leg brace with spring 36. To which category of diseases does multiple
dorsiflexion assist sclerosis belong? :D
D. spring wire short leg brace A. extrapyramidal
E. polypropylene leaf spring ankle-foot B. cerebellar
orthosis C. peripheral neuropathies
D. demyelinating
31. The main site of involvement in poliomyelitis is E. anterior horn cell
in the :C
A. dorsal roots of the spinal nerves
37. Carcinoma, especially of the lung, can produce the chance of this patient developing multiple
remote effects on the neuromuscular system. Of the sclerosis over the next 15 years? :C
following, which could be relaxed to the presence of A. less than 1%
a carcinoma? :E B. 5%
A. Myopathy C. 40%
B. Neuropathy D. 80%
C. Eaton-Lambert syndrome E. 100%
D. Only A and C
E. All of the above 44. Various types of multiple sclerosis based on the
clinical features include :E
38. Which of the following may be etiologic factors A. spinal form
in the production myelitis? :E B. cerebellar form
A. Multiple sclerosis C. cerebral form
B. Infections such as syphilis D. bulbar form
C. Acute disseminated encephalomyelitis E. all of the above
D. Spinal anesthetics
E. All of the above 45. Subacute combined degeneration of the spinal
cord is due to :D
39. After the onset of multiple sclerosis, what is the A. hereditary factors ( autosomal recessive)
usual duration of survival of the average patient? :E B. carcinomatous myelopathy
A. less than one year C. diabetes mellitus
B. 3 to 5 years D. vitamin B12 deficiency
C. 7 to 10 years E. Multiple sclerosis
D. 12 to 18 years
E. 20 years or more 46. Charcot-Marie-Tooth disease is a :A
A. form of progressive polyneuropathy
40. The IgG fraction of the gamma globulins is often B. neurological complication of diabetes
measured in the diagnostic evaluation of a patient mellitus
with suspected multiple sclerosis. In what percentage C. progressive degenerative disease of the
of cases would this parameter be abnormal at some cerebellum
time during the course of the disease? :d D. type of myopathy found in familial patterns
A. less than 5% E. none of the above
B. 20-25%
C. 45-80% 47. Werdnig-Hoffmann disease is a (an) :A
D. 75-80% A. anterior horn cell disease of infants
E. 100% B. metabolic disease resulting in mental
retardation
41. Lhemittes sign can be found in? :E C. form of peripheral neuropathy
A. Pott’s disease D. benign type of hypotonia found in infants
B. Multiple sclerosis E. disease in which the basal ganglia become
C. Cervical tumor calcified
D. Cervical chord trauma
E. All of the above 48. Upon actively flexing the head, a patient
complains of an electric shock sensation radiating
42. The most frequent motor abnormality and often down the spine into the legs. What is the name of the
the earliest manifestation of multiple sclerosis is :C sign? :D
A. isolated peripheral neuropathy A. Spurling’s sign
B. mononeuropathy B. Lasegue’s sign
C. paraplegia C. Parkinson’s sign
D. hemiplegia D. Lhermitte’s sign
E. anterior horn call damage E. Sternberg’s sign

43. A patient develops retro bulbar neuritis with 49. The primary site of pathology in Friedreich’s
decreased visual acuity and pain upon movement of ataxia is in the :C
the eye. The patient is currently 25 years old. What is A. cerebellum
B. cerebral hemispheres
C. spinal cord B. brainstem
D. peripheral nerves C. cerebral hemispheres
E. muscles D. cranial nerves
E. cranial bones
50. The most common age group for the development
of parkinson’s disease is :D
A. 5 to 15 years 57. The most common primary sites of metastatic
B. 25 to 35 years tumors to the brain are the :A
C. 40 to 50 years A. lungs and breast
D. 50 to 60 years B. prostate and stomach
E. over 60 years C. thyroid and kidney
D. uterus and bone
51. The basic mechanism for parkinsonism is :D E. colon and liver
A. calcification of the basal ganglia
B. diffuse cerebellar artery 58. The most frequent type of metastatic lesion in the
C. an abnormality of acetylcholine release in brain is :B
the thalamus A. diffuse microscopic infiltration
D. reduction in dopamine concentration in the B. multiple metastasis
caudate and putamen C. a single large metastatic lesion
E. an autosomal dominant disorder with D. A and C are equally common
delayed clinical manifestations E. A and B are equally common

52. Another name for the disorder “paralysis agitans 59. The differential diagnosis of brain tumor should
is :D include :E
A. Alzheimer’s disease A. cerebrovascular disease
B. Pick’s disease B. subdural hematoma
C. Still’s disease Parkinson’s disease C. brain abscess
D. Chronic alcoholic cerebellar degeneration D. aneurysm
E. all of the above
53. features typical of Parkinson’s disease include :E
A. rigidity of movement 60. The most frequent primary sites of metastatic
B. shuffling gait tumors to spinal cord are the :E
C. immobile face A. lungs and thyroid
D. stooped posture B. kidney and colon
E. all of the above C. uterus and liver
D. stomach and skin
54. The category of the brain tumors known as E. breast and prostate
gliomas can be differentiated into specific types of
gliomas. Which type of glioma is most common? :D 61. The most common intramedullary spinal cord
A. Glioblastoma multiforme tumors are the :C
B. Medulloblastoma A. tuberculomas
C. Epyndymoma B. meningiomas
D. Astrocytoma C. gliomas
E. Spongioblastoma polare D. lipomas
E. metastatic tumors
55. The most common type of brain tumors are
the :E 62. Lead neuritis involves chiefly the :B
A. cerebellepontine andle tumors A. sensory nerve fibers
B. pituitary tumors B. motor nerve fibers
C. metastatic tumors C. autonomic nerve fibers
D. meningiomas D. both A and C
E. gliomas E. both A and B

56. The most common location for a brain tumor in 63. Neuritis may occur following serum injection.
the adult is the :C Which is the most frequent cause of a serum neuritis?
A. posterior fossa :E
A. diphtheria antitoxin 69. A patient develops Bell’s palsy. What would be
B. scarletb fever antitoxsin the approximate chance of obtaining adequate
C. antimeningococcic serum recovery without resorting to surgery? :D
D. antineumococcic serum A. 5-10%
E. tetanus antitoxin B. 35-40%
C. 55-60%
64. Immediately following a stroke involving the D. 75-80%
cerebral artery distribution, facial weakness is noted E. 95-100%
on the hemiplegic side. A name for this facial palsy is
:E 70. A patient develops herpes zoster around the same
A. Horner’s syndrome side. This would be referred to as :B
B. Bell’s palsy A. Marcus-Gunn syndrome
C. Hunt’s syndrome B. Ramsay-Hunt syndrome
D. Romberg’s sign C. Hassin’s syndrome
E. None of the above D. Klippel-feil syndrome
E. Bell’s palsy
65. Symptoms in the thoracic outlet syndrome can be
due to compression of :D 71. A lesion of the ulnar nerve is suspected. Which of
A. vasculature structures the following movements would be expected to be
B. lymphatic structures weaker than normal? :B
C. neurological structures A. abduction of the thumb
D. both A and C B. abduction of the little finger
E. all of the above C. extension of the metacarpophalngeal joint of
the index finger
66. Following nerve injury in the axilla, which nerve D. both A and C
would generally be expected to demonstrate the most E. all of the above
rapid return of function? :C
A. Ulnar 72. A patient who was sustained a knife wound above
B. Median the elbow is examined and found to be unable to flex
C. Radial the distal interphalangeal joint of the index finger.
D. There would be no difference in recovery What is likely lesion? :C
time A. transection of the tendon of the flexor
E. There is too much variability to be able to digitorum profundus to the index of the
predict finger
B. transection of the tendon of the flexor
67. Following a peripheral nerve in jury, degeneration digitorum sublimes to the index finger
of the axons and myelin occurs but continuity of the C. injury to the median nerve
endoneurial tubes is retained. What would be the D. injury to the ulnar nerve
approximate rate of nerve regeneration for the motor E. injury to the radial nerve
fibers? :C
A. 3mm/month 73. After a prolonged period of sitting within the legs
B. 1mm/week crossed, it is noted that there is a foot drop in one of
C. 1mm/day the legs. A likely diagnosis is :A
D. 1cm/day A. injury to the peroneal nerve
E. 0.5 cm/hr B. injury to tibial nerve
C. injury to sciatic nerve
68. A major complication of Guillain-Barre syndrome D. S1 radiculopathy
is :B E. Anterior compartment syndrome
A. renal failure
B. respiratory paralysis 74. Following severance of the median nerve, there is
C. idiopathic hepatic necrosis noted to be more pink coloration than usual along the
D. cardiac valvular involvement lateral portion of the palm of the hand. Why? :D
E. cerebral nemboli A. the veins which closely accompany the
median were probably also severed, causing
arterial blood to accumulate in the hand.
B. The arteries which is closely accompany the C. extension of the shoulder
median nerve were probably also severed, D. supination of the forearm
causing venous blood to pool E. pronation of the forearm
C. The parasympathetic vasoconstrictor fibers
were interrupted 81. A patient with a peripheral nerve injury is noted
D. The symphathetic vasoconstrictor fibers to have a positive Froment’s sign. What nerve is most
were interrupted likely injured? :B
E. Edema formation causes the skin to appear A. median
B. ulnar
75. What is cause of a Volkman’s contracture? :E C. radial
A. Disuse of the affected extremity D. musculocutaneous
B. Spasticity of the muscles in the affected E. upper trunk of the brachial plexus
extremity
C. It is an idiopathic familial disorder 82. After sustaining a peripheral nerve injury a
D. It is intrinsic disorder of the dermis patient is described as having a simian hand. What
E. It is due to ischemia nerves was most likely injured? :A
A. median
76. What is a Dupuytren’s contracture :D B. ulnar
A. contracture of the Achilles tendon C. radial
B. contracture of the hip flexors following an D. musculocutaneous
above knee amputation E. upper trunk of the brachial plexus
C. contracture of elbow following a stroke 83. A patient sustains a gunshot wound in the
D. contracture of the palmar fascia infraclavicular region and later complains of
E. contracture of the skin around the neck weakness of the arm. Upon examination the only
following a burn weakness that can be detected is in flexion of the
elbow and supination of the forearm. There is noted
77. Nerves directly originating from the upper trunk to be a strip of hypesthesia along the radial border of
of the brachial plexus include the :D the forearm. What nerve was most likely injured :E
A. suprascapular nerve A. axillary
B. axillary nerve B. ulnar
C. subclavian nerve C. median
D. only A and C D. radial
E. all of the above F. musculocutaneous

78. Which of the following muscles differs in its 84. A patient was diagnosed as having meralgia
pattern of innervation from that of the other muscles paresthetica. The patient wants to know which nerve
listed? :B is affected. Your answer is the :E
A. deltoid A. obturator nerve
B. rhomboids B. sciatic nerve
C. triceps C. peroneal nerve
D. extensor carpi ulnaris D. ilioinguinal nerve
E. opponens pollicis E. lateral femoral cutaneous nerve

79. Which of the following muscles differs in its 85. A patient with a periphral nerve injury is noted to
pattern of innervation from thet of the other muscles have a “steppage gait”. There is also noted to be
listed :B weakness in the extensor of gtraet toe. Which nerve is
A> biceps brachii most likely injured? :A
B> triceps A. peroneal
C> supraspinatus B. sural
D> infraspinatus C. tibial
E> brachioradialis D. femoral
E. lateral plantar
80. With a suprascapular nerve injury, the primary
weakness noted is in :A 86. A patient with what appears to be a complete
A. external rotation of the shoulder femoral nerve lesion complais of hypesthjesia along
B. forward flexion opf the shoulder
the medial aspect of the leg. What is the reason for 90. A Babinski sign is usually an indication of
this complaint? :D corticospinal dysfunction. The characteristics of the
A. the medial cutaneous nerve was also injured Babinski sign include :E
B. the sural nerve, which is a branch from the 1. extension of the great
femoral nerve, supplies this area toe
C. the superficial peroneal nerve was also 2. flexion of the small toes
injured 3. spreading of the small
D. the saphenous nerve, which is a branch from toes
the femoral nerve, supplies this area 4. flexion of the leg
E. there is no adequate explanation inthis case.
This appears to be functional rather than an 91. Major differential factors between cerebral
organic problem thrombosis and cerebral embolism would include: E
i. the patient with embolic
Directions: For each one of the incomplete statements disease is usually younger
or questions below, One or more of the numbered ii. generalized vascular
compltions is correct. In each case select: changes are more common
A. if only 1,2, and 3 are correct with cerebral thrombosis
B. if only 1 and 3 are correc iii. hypertension and diabetes
C. if only 2 and 4 are correct are more often associated
D. if only 4 is correc with thrombosis
E. if all are correct iv. the onset of cerebral
thrombosis is generally
87. Spasticity is associated with :A slower than that of
1. an increase in muscle stretch cerebral embolism
reflexes
2. pathological reflexes 92. Common findings in the patient with cerebral
3. an increased resistance to passive thrombosis include: B
joint movement i. hypertension
4. an overall decrease in muscle tone ii. hyperlipidemia
iii. diabetes
88. Tongue deviation can occur with both peripheral iv. collagen vascular disease
and central lesions. Which of the following is (are)
correct? :E 93. Major differential factors between cerebral
1. with left hemiplegia. The toungue thrombosis and hemorrhage would include: A
will usually deviate to tye left a. loss of consciousness is more
2. withperipheral hypoglossal common with hemorrhage
dysfunction, the toungue will b. nuchal rigidity is commonly
deviate toward the side of the present with hemorrhage
lesion c. transient ischemic attacks occur
3. with a right cortical lesion. The more often before cerebral
toungue will deviate to the left thrombosis
4. atrophy and fasciculations occur d. headache is more common with
with peripheral hypoglossal cerebral thrombosis
dysfunction
94. Findings in vertebral artery disease may include:
89. Dysfunction of the Seventh cranial nerve results E
in :C a. transirnt attacks of vertigo
1. weakness of the muscles of b. clinical findings indicating
mastication dysfunction of the lower brain stem
2. loss of taste on the anterior two and upper cervical cord
thirds of the toungue c. production of symptoms by turning
3. sensory deficit on the involved side the head sharply
of the face d. impairment of consciousness
4. weakness of facial expression
95. The differentiation between pseudobulbar palsy
and bulbar palsy includes: A
a. the usual involvement by 101. Findings are often seen with posterior inferior
pseudobulbar and corticobulbar cerebellar artery occlusion include: E
trunks above the midpons a. ipsilateral loss of pain sensation
b. the usual involvement by bulbar and normal temperature in the face
palsy of the anterior horn cells b. contralateral loss of pain and
c. pseudobulbar palsy is usually temperature
vascular in origin c. ipsilateral Horner’s syndrome
d. bulbar palsy is not associated with d. ipsilateral cerebellar incoordiantion
muscle atrophy
102. Which of the following could be considered
96. Features commonly associated with thrombosis of mechanism(s) by which an embolus can pass from
the internal carotid artery include: A the systematic circulation to the brain :C
a. transient episodes of contralateral a. the embolus can pass through a
hemiparesis or hemianesthesia patent hepatorenal shunt
b. ipsilateral (monocular) blindness b. the embolus can pass through a
c. mild impairment of consciousness patent pulmonary vasculature shunt
d. weakness greater in the leg than in c. the embolus can pass through a
the arm peripheral arterio-venous
anastomosis
97. Occlusion of the main trunk of the left middle d. the embolus can pass through a
cerebral artery usually results in the following cardiac septal defect
deficits : E
a. hemianesrhesia 103. Areas of the brain most often involved by
b. expressive aphasia hypertensive cerebral hemorrhage include the: C
c. receptive aphasia a. superficial cortex
d. hemiplegia b. basal ganglia
c. brain stem
98. A patient with mild right hemiparesis is having d. internal capsule
difficulty ascending and descending stairs and asks
for your advice. You tell him: B 104. Clinical findings common with intracerebral
a. in going up the stairs, lead with the hemorrhage include: E
sound leg 1. coma
b. in going up the srairs, lead with the 2. nuchal rigidity.
involved leg 3. Convulsions
c. in going down stairs, lead with the 4. Hemiplegia
involved leg
d. in going down stairs, lead with the 105. The most common location for “berry”
sound leg aneurysms is the: A
a. internal carotid artery
99. Transient cerebral ischemic attacks (TIA): D b. anterior communicate artery
c. middle cerebral artery
1.often last longer than 24 hours d. basilar artery
are often of insidious onset
may produce permanent neurologic deficit 106. Hematomyelia is hemorrhage into the spinal
increase the risk of cerebral infarction cord. Trauma is the most common cause. Which of
the following are typical of hematomyelia? B
100. Transient ischemic attacks are often abrupt in a. most severe involvement by
onset and may present with a wide spectrum of hemorrhage is in the gray matter
findings. Which of the following could be confused around the central canal
with a transient ischemic attack? :A b. slow insidious onset
a. multiple sclerosis c. sphincter involvement of a
b. migraine neurogenic type
c. focal seizures d. no motor system involvement
d. carpal tunnel syndrome
107. Clinical findings associated with anterior spinal
artery occlusion include: A
a. paraplegia 112. Following a radial nerve injury in the
b. loss of pain and temperature midhumeral area, which of the following muscles
sensation would be expected to be weak? :B
c. atrophy of muscles at the level of 1. extensor indicis proprius
the lesion 2. triceps
d. chronic absence of the ankle jerk 3. supinator
reflexes 4. pronator teres

108. Possible causes of anterior spinal artery 113. Which of the following can occur with meralgia
thrombosis include:E paresthetica? :B
a. syphilis 1. hypesthesia in the anterior lateral aspect of thigh
b. aortic occlusion 2. diminished of the knee-jerk reflex
c. cervical spondylosis 3. hyperesthesia in the anterior lateral aspect of the
d. arterities secondary to collagen- thigh
vascular disease 4. weakness of quadriceps

109. A patient awakens in the morning with weakness 114. Poliomyelitis should be differentiated from
in the right arm. Upon examination there is complete Guillain- Barre syndrome. Which of the following
loss of function in the wrist extensors and in the might be helpful for this purpose? : A
extensors of the fingers at the metacarpophalangeal 1. the protein content of the spinal fluid is increased
joints. There also appears to be some weakness of the with Guillain-Barre syndrome
finger abductors when they are tested in an 2. an accurate immunization history
unsupported position. What lesions are most likely 3. the cell court in the spinal fluid is usually elevated
present? D with poliomylelitis
a. median nerve 4. a flu-like illness occurs before the Guillain-Barre
b. ulnar nerve syndrome only
c. axillary nerve
d. radial nerve 115. Which of the following are typical of subacute
combined degeneration of the spinal cord? : E
110. Which statement(s) is (are) true about sweating 1. paresthesias most commonly around the feet
following a nerve injury? C 2. ataxia
a. when peripheral nerve is totally 3. impairment of the position and vibration sensation
severed, over secretion of sweat 4. spasticity due to pyramidal tract involvement
occurs in it’s area of distribution
b. an increase tendency to sweat is 116. Progressive spinal muscular atrophy
frequently seen following partial pathologically involves the :D
nerve injuries 1. extrapyramidal system
c. the area of sweating abnormality 2. corticospinal tract
following nerve section is generally 3. cerebellar tracts
much smaller than the area of 4. anterior horn cells
sensory deficit
d. when a peripheral nerve is 117. Common symptoms in amyotrophic lateral
completely sectioned, sweating no sclerosis (ASL) include : A
longer occurs in its area of 1. weakness of the hands and arms
distribution 2. muscle cramps in the legs
111. The axillary nerve in a patient was injured where 3. difficulty in talking and swallowing
it passes posteriorly around the surgical neck of the 4. numbness of the feet
humerus. What Muscle (s) would be expected to be
weak? :D 118. Amyotrophic lateral sclerosis pathologically
1. triceps involves in :C
2. teres minor 1. extrapyramidal system
3. supraspinatus 2. corticospinal tracts
4. deltoid 3. cerebellar tracts
4. anterior horn cells
119. Hypertrophic interstitial neuropathy (dejerine 4. it is generally present while the patient is at rest
scotts disease) is characterized by :E
1. weakness of the hands and feet with distal atrophy 126. Which of the following is (are) true about
2. familial inheritance paralysis agitans? :C
3. loss of muscle stretch reflexes 1. tremor is often the initial symptom
4. enlargement of peripheral nerves 2. handwriting is characterized by the abnormalities
of macrographia
120. Findings typical of Charlot –Marie –tooth 3. slowness of movement is common
disease include :A 4. the onset is often rather abrupt
1. gradual onset of symptoms
2. atrophy and weakness of leg muscles 127. Which of the following is (are) true about
3. impaired superficial sensation in the distal portions astroeytomas? :E
of the limbs 1. they are the most benign type of glioma
4. relatively normal nerve conduction studies 2. they grow very slowly
3. they occur in both solid and cystic forms
121. The characteristics of progressive bulbar palsy 4. the prognosis is better for cerebellar rather than
include :E cerebral astroesytomas
1. pathologic involvement of cranial nerve motor
nuclei 128. Which of the following is (are) true about
2. usually symmetry in its involvement glioblastoma multiforme brain tumors? :A
3. difficulty in articulation is a common early 1. they are highly malignant and rapidly growing
symptom 2. they usually occur in persons over the age of 35
4. overall prognosis is poor years
3.the prognosis for survival is generally poor
122. Kugelberg-Welander disease is a familial 4. most are located in the cerebellum
disorder involving anterior horn cells. Which of the
following is (are) characteristic of this disoreder? :B 129. Which of the following is (are) true about
1. onset between the ages of 2 and 17 years medulloblastomas? :C
2. corticospinal tract involvement 1. they are most common in the cerebral hemisphere
3. weakness and atrophy beginning in the proximal 2. headache, vomiting and ocular distrurbances are
limb muscles common
4. relatively rapidly progressive course with death 3. they occur primarily in adults
usually around the age of 30 years 4. surgical removal and intensive irradiation are the
best forms of treatment
123. characteristics of infantile progressive muscular
atrophy (werdnig-hoffmann disease) include :A 130. Which of the following is (are) true about
1. usual development of symptoms in the first year of meningiomas of the brain? A
life 1. they are the most benign of all brain tumors
2. flaccid weakness which makes the infant nearly 2. they may be multiple
immobile 3. they are found chiefly in adults
3. joint hyperextensibility 4. they are best treated with irradiation and
4. death usually around the age of 20 years chemotherapy

124. Which of the following is (are) true about 131. Which of the following is (are) true about spinal
Friedreich’s ataxia? :E cord tumors? C
1. initial symptoms generally develop in the 5 to 20 1. they are equal in frequency to brain tumors
year age group 2. they are most frequently extramedullary rather than
2. the disorder is hereditary intramedullary
3. ataxia is usually the first symptom 3. they most often occur in children
4. kyphoscoliosis is common 4. they are most commonly fibroblastomas

125. Which of the following is (are) true about the 132. Which of the following is (are) true about
tremor found in parkinson’s disease : C fibroblastomas type of spinal cord tumor?E
1. it is accentuated by movement 1. most are found in the thoracic portion of the
2. it is diminished with movement vertebral column
3. it disappear while the patient is at rest
2. the spinal cord is compressed and pushed to the Directions: the numbered items should be answered
side to the following key:
3. it is extramedullary in location A. if only A is correct
4. pain is common symptom B. if only B is correct
C. if both A & B are correct
133. Which of the following is (are) true about D. if neither A nor B are is correct
syringomyelia? B
1. it is characterized by cavity formation in the spinal Question 139 through 144
cord 139. sudden severe headache :B
2. it generally presents symptomatically over the age 140. loss of consciousness :B
of 50 years 141. hypertension C
3. it most commonly involves the cervical portion of 142. often develops during sleep A
the spinal cord 143. bloody cerebrospinal fluid B
4. it often presents with rapid development of 144. nuchal rigidity B
symptoms A. cerebral thrombosis
B. cerebral hemorrhage
134. Which of the following is (are) true about the C. both A and B
Arnold-Chian malformation? A D. neither A nor B
1. it is congenital malformation involving the
cerebellum and lower brain stem Questions 145 through 151
2. it is commonly presents in children with 145. often related to he presence B
myelomenigolece 146. frequent involvement of the ocular muscles A
3. symptoms of increased intracranial pressure may 147. usually a good response to anticholinesterase
occur medications A
4. it is usually a benign disorder which requires no 148. characterized by muscular weakness and fatigue
treatment C
149. repeated activity results in an increase in
135.Characteristic of alcoholic neuritis include E weakness A
1. its general occurrence in chronic rather than 150. repeated activity results in an increase in
episodic drinker strength B
2. may develop quite acutely 151. may be associated with neoplasm of the thymus
3. pain is prominent A
4. a probably greater relationship to avitaminosis A. myasthenia gravis
than to the alcohol itself B. myasthenia syndrome
C. both A & B
136. characteristics of diabetic neuritis include B D. neither A nor B
1. pain, especially at night
2. symptoms developing Question 152 through 155
3. in younger diabetic patient 152. a fairy common complication is the carpal
4. diminished muscle stretch reflexes tunnel syndrome B
5. severe muscle atrophy 153. a myopathy may be present C
154. a fine rapid tremor is often present A
137. characteristic of arsenic neuritis include :A 155. can be associated with muscle wasting A
1. symmetric polyneuritis A. hyperthyroidism
2. ataxia B. hypothyroidism
3. transverse while lines on the fingernails C. both A & B
4. painless sensory deficit in the feet D. neither A nor B

138. characteristic of the Guillain-Barre syndrome Questions 156 through 160


include :A 156. weakness is on the ipsilateral side of the lesion
1. clinical onset is usually preceded by recent A
respiratory or gastrointestinal infection 157. weakness is on the contralateral side of the
2. weakness is symmetrical lesion B
3. sensory abnormalities are usually minimal 158. the upper half of the face is involved to a greater
4. pleocytosis is a common spinal fluid finding extent than is the lower half D
159. the lower half of the face is involved to a greater
extent than is the upper half B
160. the entire half of the face is involved A
A. peripheral nerve injury to the seventh cranial
nerve
B. alteration of facial function due to a supranuclear
lesion
C. both A & B
D. neither A nor B

Question 161 through 165


161. often gives more difficulty with dressing
activities B
162. often is accompanied with aphasia A
163. visual perpectual problems are relatively
common B
164. usually requires longer rehabilitation programs
before ambulation B
165. rehabilitation programs are often beneficial C
A. right hemiplegia
B. left hemiplegia
C. both A & B
D. neither A nor B

Anda mungkin juga menyukai